Final

Pataasin ang iyong marka sa homework at exams ngayon gamit ang Quizwiz!

caregiver

help patients maintain and regain health, manage disease and symptoms, and attain a maximal level of function and independence through the healing process

health promotion

helps individuals maintain or enhance their present health

Emic worldview

insider perspective

Short Assessment of Health Literacy - Spanish and English (SAHL-S & E)

instrument consisting of comparable tests in English and Spanish with good reliability and validity in both languages.

identity

internal sense of individuality, wholeness, and consistency of a person over time and in different situations

Connectedness

intrapersonally (connected within oneself), interpersonally (connected with others and the environment) transpersonally (connected with the unseen, God or a higher power)

Culture

learned and shared beliefs, values, norms, and traditions of a particular group.

cultural awareness

process of conducting a self-examination of one's own biases towards other cultures and the in-depth exploration of one's cultural and professional background

advocate

protect your patient's human and legal rights and provide assistance in asserting these rights.

provider of care

provide direct, hands on care in all health care agencies and settings, they have an active role in: illness prevention, health promotion, health maintenance

support systems

provide patients with greatest sense of well-being and serve as a human link, connecting patient, nurse, and patient's lifestyle before an illness

life satisfaction

satisfied with their life and how they are using their abilities

Maslow's Hierarchy of Needs

(level 1) Physiological Needs, (level 2) Safety and Security (level 3) Love and Belonging (level 4) Self Esteem (level 5) Self Actualization

Evaluation

- Through the patient's eyes - Patient's perceived success in meeting goals and outcomes. - Patient outcomes

Three levels of prevention

1. Primary 2. Secondary 3. Tertiary

Self-Concept Nursing Diagnoses

1. disturbed body image 2. disturbed personal identity 3. impaired role performance 4. situational low self-esteem

33. A nurse is caring for a patient who fell on the ice and has connective tissue damage in the wrist and hand. The patient describes the pain as throbbing. Which type of pain does the nurse document in this patient's medical record? 1. Visceral pain 2. Somatic pain 3. Centrally generated pain 4. Peripherally generated pain

2. Somatic pain

15. A nurse is assessing a patient who began experiencing severe pain 3 days ago. When the nurse asks the patient to describe the pain, the patient states, "The pain feels like it is in my stomach. It is a burning pain, and it spreads out in a circle around the spot where it hurts the most." Which type of pain does the nurse document the patient is having at this time? 1. Superficial pain 2. Idiopathic pain 3. Chronic pain 4. Visceral pain

4. Visceral pain

Rapid Estimate of Adult Literacy in Medicine-Short Form (REALM-SF)

7 item word recognition test to provide clinicians with a valid quick assessment of patient health literacy

love and belonging needs

A person's need to have affectionate relationships with people and to have a place in a group

developmental stage

A person's perceptions of health, illness, and health behaviors change over time

A nurse is preparing to make a consult. In which order, beginning with the first step, will the nurse take? 1. Identify the problem. 2. Discuss the findings and recommendation. 3. Provide the consultant with relevant information about the problem. 4. Contact the right professional, with the appropriate knowledge and expertise. 5. Avoid bias by not providing a lot of information based on opinion to the consultant. A. 1, 4, 3, 5, 2 B. 4, 1, 3, 2, 5 C. 1, 4, 5, 3, 2 D. 4, 3, 1, 5, 2

A. 1, 4, 3, 5, 2

A 17-year-old patient, dying of heart failure, wants to have organs removed for transplantation after death. Which action by the nurse is correct? A. Instruct the patient to talk with parents about the desire to donate organs. B. Notify the health care provider about the patient's desire to donate organs. C. Prepare the organ donation form for the patient to sign while still oriented. D. Contact the United Network for Organ Sharing after talking with the patient.

A. Instruct the patient to talk with parents about the desire to donate organs.

The nurse is interviewing a patient with a hearing deficit. Which area should the nurse use to conduct this interview? A. The patient's room with the door closed B. The waiting area with the television turned off C. The patient's room before administration of pain medication D. The waiting room while the occupational therapist is working on leg exercises

A. The patient's room with the door closed

1. Levothyroxine is available in 88-mcg tablet form. Convert this dose to milligram strength. (do not round)

ANS: 0.088 mg

1. When giving medications, the nurse will follow the rights of medication administration. The rights include the right documentation, the right reason, the right response, and the patient's right to refuse. Which of these are additional rights? (Select all that apply.) 1. Right drug 2. Right route 3. Right dose 4. Right diagnosis 5. Right time 6. Right patient

ANS: 1, 2, 3, 5, 6

2. Digoxin is available in 0.25-mg tablet form. Convert this dose to microgram strength. (do not round)

ANS: 250 mcg

1. A drug dose that delivers 10 mg has a half-life of 5 hours. Identify how much drug will remain in the body after one half-life.

ANS: 5 mg

2. Which nursing actions will the nurse perform in the evaluation phase of the nursing process? (Select all that apply.) A. Set priorities for patient care. B. Determine whether outcomes or standards are met. C. Ambulate patient 25 feet in the hallway. D. Document results of goal achievement. E. Use self-reflection and correct errors.

ANS: B, D, E

Body image stressors

Affect appearance, structure or function of a body part

Certified Nurse-Midwife (CNM)

An APRN who is also educated in midwifery and is certified by the American College of Nurse-Midwifes

A nurse used spiritual rituals as an intervention in a patient's care. Which of the following questions is most appropriate to evaluate its efficacy? 1. Do you feel the need to forgive your wife over your loss? 2. What can I do to help you feel more at peace? 3. Did either prayer or meditation prove helpful to you? 4. Should we plan on having your family try to visit you more often in the hospital?

Answer 3: Rituals include participation in worship, prayer, sacraments (e.g., baptism, Holy Eucharist), fasting, singing, meditating, scripture reading, and making offerings or sacrifices. When you include the use of rituals in a patient's plan of care, evaluate whether the patient perceived these activities as useful. If not, other interventions will be necessary.

The nurse is intervening for a family member with role strain. Which direct care nursing intervention is most appropriate? A. Assisting with activities of daily living B. Counseling about respite care options C. Teaching range-of-motion exercises D. Consulting with a social worker

B. Counseling about respite care options

2. You are in the process of admitting an ethnically diverse patient. To plan culturally competent care, you will conduct a cultural assessment that includes: A. biocultural history. B. ethnohistory. C. negotiation. D. ethnocentrism.

B. Ethnohistory

A nurse is getting ready to discharge a patient who is experiencing impaired physical mobility. What does the nurse need to do before discontinuing the patient's plan of care? A. Determine whether the patient has transportation to get home. B. Evaluate whether patient goals and outcomes have been met. C. Establish whether the patient has a follow-up appointment scheduled. D. Ensure that the patient's prescriptions have been filled to take home.

B. Evaluate whether patient goals and outcomes have been met.

A nurse is using the problem-focused approach to data collection. Which action will the nurse take first? A. Completing the questions in chronological order B. Focusing on the patient's presenting situation C. Making accurate interpretations of the data D. Conducting an observational overview

B. Focusing on the patient's presenting situation

A nurse is completing a care plan. Which intervention is most appropriate for the nursing diagnostic statement Impaired skin integrity related to shearing forces? A. Administer pain medication every 4 hours as needed. B. Turn the patient every 2 hours, even hours. C. Monitor vital signs, especially rhythm. D. Keep the bed side rails up at all times.

B. Turn the patient every 2 hours, even hours.

A young male patient is diagnosed with testicular cancer. Which action will the nurse take first? A. Provide information to the patient. B. Allow time for the patient's friends. C. Ask about the patient's priority needs. D. Find support for the family and patient.

C. Ask about the patient's priority needs.

A nurse is using assessment data gathered about a patient and combining critical thinking to develop a nursing diagnosis. What phrase is used to identify what the nurse is doing? A. Assigning clinical cues B. Defining characteristics C. Diagnostic reasoning D. Diagnostic labeling

C. Diagnostic reasoning

A nurse performs an assessment on a patient. Which assessment data will the nurse use as an etiology for Acute pain? A. Discomfort while changing position B. Reports pain as a 7 on a 0 to 10 scale C. Disruption of tissue integrity D. Dull headache

C. Disruption of tissue integrity

A patient with a spinal cord injury is seeking to enhance urinary elimination abilities by learning self-catheterization versus assisted catheterization by home health nurses and family members. The nurse adds Readiness for enhanced urinary elimination in the care plan. Which type of diagnosis did the nurse write? A. Risk B. Problem focused C. Health promotion D. Collaborative problem

C. Health promotion

A severely depressed patient cannot state any positive attributes to life. The nurse patiently sits with this patient and assists the patient to identify several activities the patient is actually looking forward to in life. Which spiritual concept is the nurse trying to promote? A. Time management B. Reminiscence C. Hope D. Faith

C. Hope

An obstetric nurse comes across an automobile accident. The driver seems to have a crushed upper airway, and while waiting for emergency medical services to arrive, the nurse makes a cut in the trachea and inserts a straw from a purse to provide an airway. The patient survives and has a permanent problem with vocal cords, making it difficult to talk. Which statement is true regarding the nurse's performance? A. The nurse acted appropriately and saved the patient's life. B. The nurse stayed within the guidelines of the Good Samaritan Law. C. The nurse took actions beyond those that are standard and appropriate. D. The nurse should have just stayed with the patient and waited for help.

C. The nurse took actions beyond those that are standard and appropriate.

Family members gather in the emergency department after learning that a family member was involved in a motor vehicle accident. After learning of the family member's unexpected death, the surviving family members begin to cry and scream in despair. Which phase does the nurse determine the family is in according to the Attachment Theory? A. Numbing B. Reorganization C. Yearning and searching D. Disorganization and despair

C. Yearning and searching

A nurse manager conducts rounds on the unit and discovers that expired stock medicine is still in the cabinet despite the e-mail that was sent stating that it had to be discarded. The staff nurse dress code is not being adhered to as requested in the same e-mail. Several staff nurses deny having received the e-mail. Which action should the nurse manager take? A. Close the staff lounge. B. Enforce a stricter dress code. C. Include the findings on each staff member's annual evaluation. D. Place a hard copy of announcements and unit policies in each staff member's mailbox.

D. Place a hard copy of announcements and unit policies in each staff member's mailbox.

A nurse facilitates the transplant coordinator in make a request for organ and tissue donation from the patient's family. What is the primary rationale for the nurse's action? A. The nurse is not as knowledgeable as the coordinator. B. The nurse is uncomfortable asking the family. C. The nurse does not want to upset the family. D. The nurse is following a federal law.

D. The nurse is following a federal law.

A terminally ill patient is experiencing constipation secondary to pain medication. Which is the best method for the nurse to improve the patient's constipation problem? A. Contact the health care provider to discontinue pain medication. B. Administer enemas twice daily for 7 days. C. Massage the patient's abdomen. D. Use a laxative.

D. Use a laxative.

Communication

Effective communication strategies are fundamental to providing quality care, coordinating and managing patient care. Uses verbal and non-verbal skills

Florence Nightingale

Established sanitary nursing care units. Founder of modern nursing. began professional education of nursing.

educator

Explains, demonstrates, reinforces, and evaluates the patient's progress in learning.

Acquiring Cultural Knowledge

HCP Must focus on three specific issues. 1. Health related beliefs and cultural values 2. Care Practices 3. Disease incidence and prevalance

social determinants of health

Include a variety of social, commercial, cultural, economic, environmental, and political factors that affect health inequalities.

negative health behaviors

Inculde practices actually or potentially harmful to health such as smoking, drug or alcohol abuse, poor diet and refusal to take necessary medications.

Native Americans

Individual tribal beliefs influence food practices.

Emotional Development

Influence personal beliefs about health-related matters.

body image

Involves attitudes related to the body including physical appearance, structure, or function

LEARN model

L: listen (tell me more, I understand) E: explain (your perception of the problem) A: acknowledge (recognize the differences and build on similarities) R: recommend(must involve patient) N: negotiate(treatment plan, incorporate patients culture into the plan)

Models of Health and Illness: Health Belief Model

Models help explain complex conceptd or ideas, such as health and illness. They are sometimes based on facts or misinformation, common sense or myths, goof or bad experiences, or reality or false expectations.

Potential diagnoses

Risk for Spiritual Distress Defective Spiritual Distress Hopelessness Spiritual Distress: anger towards god, lack of hope/meaning Decreased Spiritual Distress:

ritual and practice

Rituals include participation in worship, prayer, sacraments, fasting, singing, meditating, scripture reading, and making offerings or sacrifices.

Spiritual factors

Spirituality is reflected in how a person lives his or her life, including the values and beliefs exercised, the relationships established with family and friends, and the ability to find hope and meaning in life.

Holistic Health Model

The *holistic health model* assesses the whole person because it views the mind, body, and spirit as interdependent and functioning as a whole within the environment. Health depends on all these factors working together.

Transcendence

The belief that there is a force out side of and greater than the person exists beyond the material world

spiritual distress

a disruption in the life principle that pervades a person's entire being and transcends the person's biologic and psychosocial nature

cultural identity

a feeling of belonging to a group that shares the same culture, or way of life

5. A patient with a new prescription for a diuretic has just reviewed with the nurse how to include more potassium in her diet. This reflects learning in which domain? a. Cognitive b. Affective c. Physical d. Psychomotor

a. Cognitive

Mormonism

abstain from alcohol and caffeine

life and self responsibility

ask about a patient's understanding of illness limitations or threats and how the patient will adjust

stereotype

assumed belief regarding a particular group

Jehovah's Witness

avoid food prepared with or containing blood

implicit bias

aware of the bias that is present. we take full responsibility for and must recognize and acknowledge our actions

14. A nurse is providing home care to a home-bound patient treated with intravenous (IV) therapy and enteral nutrition. What is the home health nurse's primary objective? a. Screening b. Education c. Dependence d. Counseling

b. Education

The nurse establishes trust and talks with a school-aged patient before administering an injection. Which type of implementation skill is the nurse using? a. Cognitive b. Interpersonal c. Psychomotor d. Judgmental

b. Interpersonal

The nurse has become aware of missing narcotics in the patient care area. Which ethical principle obligates the nurse to report the missing medications? a. Advocacy b. Responsibility c. Confidentiality d. Accountability

b. Responsibility

The patient appears to be in no apparent distress, but vital signs taken by assistive personnel reveal an extremely low pulse. The nurse then auscultates an apical pulse and asks the patient whether there is any history of heart problems. The nurse is utilizing which critical thinking skill? a. Evaluation b. Explanation c. Interpretation d. Self-regulation

c. Interpretation

The nurse completes a thorough assessment of a patient and analyzes the data to identify nursing diagnoses. Which step will the nurse take next in the nursing process? a. Assessment b. Diagnosis c. Planning d. Implementation

c. Planning

The nurse inserts an intravenous (IV) catheter using the correct technique and following the recommended steps according to standards of care and hospital policy. Which type of implementation skill is the nurse using? a. Cognitive b. Interpersonal c. Psychomotor d. Judgmental

c. Psychomotor

Social determinants

conditions in which people are born, grow, live, work and age

Nurse Researcher

conducts evidence-based practice and research to improve nursing care and further define and expand the scope of nursing practice. Masters degree preferred

Storytelling

conveys culture, combining personal experience with the commonalities of all human experiences

A staff nurse delegates a task to a nursing assistive personnel (NAP), knowing that the NAP has never performed the task before. As a result, the patient is injured, and the nurse defensively states that the NAP should have known how to perform such a simple task. Which element of the decision-making process is the nurse lacking? a. Authority b. Autonomy c. Responsibility d. Accountability

d. Accountability

A registered nurse administers pain medication to a patient suffering from fractured ribs. Which type of nursing intervention is this nurse implementing? a. Collaborative b. Independent c. Interdependent d. Dependent

d. Dependent

3. You are assigned to care for a patient who retired 6 months ago. While providing care, you identify that this patient is struggling emotionally with change. This situation is most likely associated with the self-concept component of: A. identity stressor. B. sexuality stressor. C. body image stressor. D. role-performance stressor.

d. role performance stressor

cultural encounter

encourages HCP to directly engage in face-to-face cultural interactions and other types of encounters with clients from culturally diverse background in order to modify existing beliefs about a cultural group to prevent stereotyping

hope

energizing source that has an orientation to future goals and outcomes

vocation

expression of spirituality showing appreciation for life in the variety of things people do

Russian Orthodox Church

fast days and "no meat" rule

Prayer

gives opportunity to renew personal faith

self-esteem needs

need to feel good about oneself and to believe others hold one in high regard

Buddhism

no alcohol use , fast on holy days

agnostic

no known ultimate reality

islam

no pork and alcohol

Hinduism

not to kill any living creature

measures that support self-esteem

nutrition, adequate sleep, exercise, stress management

racial identity

one's self-identification with one or more social groups in which a common heritage with a particular racial group is shared

safety and security needs

person's need to be protected from actual or potential harm and to have freedom from fear

Maslow's Hierarchy

physiological, safety, love/belonging, esteem, self-actualization

supporting rituals

plan care to allow time for religious readings, spiritual visitations, or attendance at religious services

unconscious bias

refers to a bias we are unaware of, & that happens outside of our control, which is influenced by our personal background, cultural environment, & personal experiences.

Health belief model first component

the individual's preception of susceptibility to an illness

self-actualization needs

the need to be the best one can be; at the top of Maslow's hierarchy

emotional factors

the patient's degree of stress, depression, or fear can influence health beliefs and practices

In which order will the nurse use the nursing process steps during the clinical decision-making process? 1. Evaluating goals 2. Assessing patient needs 3. Planning priorities of care 4. Determining nursing diagnoses 5. Implementing nursing interventions A. 2, 4, 3, 5, 1 B. 4, 3, 2, 1, 5 C. 1, 2, 4, 5, 3 D. 5, 1, 2, 3, 4

A. 2, 4, 3, 5, 1

Which action indicates the nurse is using a PICOT question to improve care for a patient? A. Practices nursing based on the evidence presented in court. B. Implements interventions based on scientific research. C. Uses standardized care plans for all patients. D. Plans care based on tradition.

B. Implements interventions based on scientific research.

A preadolescent patient is experiencing maturational stress. Which area will the nurse focus on when planning care? A. Identity issues B. Self-esteem issues C. Physical appearance D. Major changing life events

B. Self-esteem issues

Which action should the nurse take to best develop critical thinking skills? A. Study 3 hours more each night. B. Attend all in-service opportunities. C. Actively participate in clinical experiences. D. Interview staff nurses about their nursing experiences.

C. Actively participate in clinical experiences.

illness prevention

activities such as immunization programs protect patients from actual or potential threats to health

Clinical Nurse Specialist (CNS)

an APRN who is an expert clinician in a specialized area of practice

terminal illness

an illness or injury from which the person will not likely recover - spiritual distress

health literacy

the degree to which individuals have the capacity to obtain, process, and understand basic health information and services needed to make appropriate health decisions

Diet therapies

food and nutrition are important aspects of patient care and often an important component of religious observances

health education

includes topics such as physical awareness, stress management, and self-responsibility

Autonomy and Accountability

independent nursing interventions and implementation with responsibility and accountability for actions taken

Assimilation

individual adapts to host's cultural values and no longer prefers the components of the origin culture.

Acculturation

individual or group transitions from one culture and develops traits of another culture

Health belief model second component

individual's perception of the seriousness of the illness. Influenced and modified by demographic and sociophsychological variables and cues to action

self-esteem

individuals overall feeling of self-worth or the emotional appraisal of self-concept

Nursing Administration

manages patient care and the delivery of specific nursing services within a health care agency

near-death experience (NDE)

psychological phenomenon close to clinical death or recovered after declared death

Standards of Professional Nursing Practice

-Contain authoritative statements of the duties that all RN, regardless of role, population, or specialty are expected to perform competently. -Duties are supported by a critical thinking model known as the nursing process

ANA Standards of Professional Performance

-Describes a competent level of behavior in the professional role. - The standards provide a method to assure the patients they are receiving high- quality care and to ensure nurses must know exactly what is necessary to provide nursing care.

Which sociocultural finding in the history of a patient will alert the nurse to a possible developmental problem? A. Family relocation B. Childhood obesity C. Prolonged poverty D. Loss of stamina

C. Prolonged poverty

A patient in a motor vehicle accident states, "I did not run the red light," despite very clear evidence on the street surveillance tape. Which defense mechanism is the patient using? a. Denial b. Conversion c. Dissociation d. Compensation

a. Denial

Spiritual Well-Being Scale (SWB)

has 20 questions that assess a patient's relationship with God and his or her sense of life purpose and life satisfaction

Marginalised groups

have poorer outcomes because of complex interactions due to behaviors, environment, and clinical care received.

cultural knowledge

healthcare professional seeks and obtains a sounud educational base about culturally diverse groups.

fellowship and community

kind of relationship that an individual has with other people

Orem's Theory

known as self-care deficit theory which focuses on the patient's self care needs

Judaism

kosher diet retrictions - avoid pork and shellfish

cultural desire

motivation of HCP to "want to" engage in the process of being culturally aware, culturally knowledgeable, culturally skillful

cultural competence

must be culturally sensitive, culturally appropriate, and culturally competent to meet to meet the multifaceted needs of the patient.

Nurse Practitioner (NP)

An APRN who provides health care to a group of patients, usually in an outpatient, ambulatory care or community-based setting

Certified Registered Nurse Anesthetist (CRNA)

An APRN with advanced education in a nurse anesthetia accredited program

faith, belief

- Ask about a religious source of guidance. - Understand the patient's philosophy of life.

Collecting a patient history

-Linguistic Competence -Provide language assistance resources -Inform all of the availability of language assistance -Ensure competence of those providing language assistance -Provide print/multimedia materials in local languages

2. A member of an investigational drug study team is working with healthy volunteers whose participation will help to determine the optimal dosage range and pharmacokinetics of the drug. The team member is participating in what type of study? 1. Phase I 2. Phase II 3. Phase III 4. Phase IV

1. Phase I

Professional Responsibilities and Roles

-You are responsible for obtaining and maintaining specific knowledge and skills for a variety of professional roles and responsibilities. -Nurses concern for meeting their patients needs remains the same whether care focuses on health promotion and illness prevention, diseases and symptom management, family support or end-of-life-care.

8. A nurse wants to find all the pertinent patient information in one record, regardless of the number of times the patient entered the health care system. Which record should the nurse access? 1. Electronic medical record 2. Electronic health record 3. Electronic charting record 4. Electronic problem record

2. Electronic health record

2. A nurse is describing the purposes of a health care record to a group of nursing students. Which purposes will the nurse include in the teaching session? (Select all that apply.) 1. Communication 2. Legal documentation 3. Reimbursement 4. Nursing process 5. Research 6. Education

1-7

7. In order to receive payment for care provided, nursing centers must comply with requirements outlined in what federal legislation? 1. Omnibus Budget Reconciliation Act 2. Medicare Act 3. Medicaid Act 4. Affordable Care Act

1. Omnibus Budget Reconciliation Act

5. Which activity best reflects the implementation phase of the nursing process for the patient who is newly diagnosed with hypertension? 1. Providing education on keeping a journal of blood pressure readings 2. Setting goals and outcome criteria with the patient's input 3. Recording a drug history regarding over-the-counter medications used at home 4. Formulating human needs statements regarding deficient knowledge related to the new treatment regimen

1. Providing education on keeping a journal of blood pressure readings

A patient who is depressed is crying and verbalizes feelings of low self-esteem and self-worth, such as "I'm such a failure ... I can't do anything right." What is the nurse's best response? 1. Remain with the patient until he or she validates feeling more stable. 2. Tell the patient that is not true and that every person has a purpose in life. 3. Review recent behaviors or accomplishments that demonstrate skill ability. 4. Reassure the patient that you know how he or she is feeling and that things will get better.

1. Remain with the patient until he or she validates feeling more stable. Demonstrating acceptance of the patient by supportively sitting with him or her builds a therapeutic nurse-patient relationship. The nurse's presence signals value and allows the patient to explore issues of self-concept and self-esteem. In contrast, giving false hope is neither therapeutic nor conveys acceptance, while focusing on skill ability signals conditional approval.

3. A nurse is developing a plan to reduce data entry errors and maintain confidentiality. Which guidelines should the nurse include? (Select all that apply.) 1. Bypass the firewall. 2. Implement an automatic sign-off. 3. Create a password with just letters. 4. Use a programmed speed-dial key when faxing. 5. Impose disciplinary actions for inappropriate access. 6. Shred papers containing personal health information (PHI). ANS: B, D, E, F

2,4,5,6

9. A nurse is teaching a patient about patient-controlled analgesia (PCA). Which statement made by the patient indicates to the nurse that teaching is effective? 1. "I will only need to be on this pain medication." 2. "I feel less anxiety about the possibility of overdosing." 3. "I can receive the pain medication as frequently as I need to." 4. "I need the nurse to notify me when it is time for another dose."

2. "I feel less anxiety about the possibility of overdosing."

17. The nurse is caring for a patient and is focusing on modifiable factors that contribute to pain. Which areas does the nurse focus on with this patient? 1. Age and gender 2. Anxiety and fear 3. Culture and ethnicity 4. Previous pain experiences and cognitive abilities

2. Anxiety and fear

6. Which action by a novice nurse will cause the preceptor to provide follow up instructions? 1. Documents descriptively. 2. Charts consecutively on every other line. 3. Ends each entry with signature and title. 4. Uses quotations to note patients' exact words.

2. Charts consecutively on every other line.

9. An 83-year-old woman has been given a thiazide diuretic to treat heart failure. She and her caregiver should be told to watch for which problems? 1. Constipation and anorexia 2. Fatigue, leg cramps, and dehydration 3. Daytime sedation and lethargy 4. Edema, nausea, and blurred vision

2. Fatigue, leg cramps, and dehydration

34. The nurse is caring for an infant in the intensive care unit. Which information should the nurse consider when planning care for this patient? 1. Infants cannot be assessed for pain. 2. Infants respond behaviorally and physiologically to painful stimuli. 3. cannot tolerate analgesics owing to an underdeveloped metabolism. 4. Infants Infants have a decreased sensitivity to pain when compared with older children.

2. Infants respond behaviorally and physiologically to painful stimuli.

8. The nurse is reviewing facts about pharmacology for a review course. The term legend drug refers to which item? 1. Over-the-counter drugs 2. Prescription drugs 3. Orphan drugs 4. Older drugs

2. Prescription drugs

6. A nurse provides immunization to children and adults through the public health department. Which type of health care is the nurse providing? 1. Primary care 2. Preventive care 3. Restorative care 4. Continuing care

2. Preventive care

14. A nurse developed the following discharge summary sheet. Which critical information should the nurse add? TOPICDISCHARGE SUMMARY Medication Diet Activity level Follow-up care Wound care Phone numbers When to call the doctor Time of discharge 1. Clinical decision support system 2. Admission nursing history 3. Mode of transportation 4. SOAP notes

3. Mode of transportation

4. A nurse is caring for a patient in the hospital. When should the nurse begin discharge planning? 1. When the patient is ready. 2. Close to the time of discharge. 3. Upon admission to the hospital. 4. After an order is written/prescribed.

3. Upon admission to the hospital.

1. Which behaviors indicate the student nurse has a good understanding of confidentiality and the Health Insurance Portability and Accountability Act (HIPAA)? (Select all that apply.) 1. Writes the patient's room number and date of birth on a paper for school. 2. Prints/copies material from the patient's health record for a graded care plan. 3. Reviews assigned patient's record and another unassigned patient's record. 4. Gives a change-of-shift report to the oncoming nurse about the patient. 5. Reads the progress notes of assigned patient's record. 6. Discusses patient care with the hospital volunteer.

4,5

30. The nurse is caring for a patient who suddenly experiences chest pain. What is the nurse's first priority? 1. Call the rapid response team. 2. Start an intravenous (IV) line. 3. Administer pain-relief medications. 4. Ask the patient to rate and describe the pain.

4. Ask the patient to rate and describe the pain.

3. The nurse has been monitoring the patient's progress on a new drug regimen since the first dose and documenting the patient's therapeutic response to the medication. Which phase of the nursing process do these actions illustrate? 1. Human needs statement 2. Planning 3. Implementation 4. Evaluation

4. Evaluation

4. A patient has been selected as a potential recipient of an experimental drug for multiple sclerosis. The nurse knows that when informed consent has been obtained, it indicates which of these? 1. The patient has been informed that he or she will need to stay in the study until it ends. 2. The patient will be informed of the details of the study as the research continues. 3. The patient will receive the actual drug during the experiment. 4. The patient has had the study's purpose, procedures, and possible benefits as well as risks involved explained to him.

4. The patient has had the study's purpose, procedures, and possible benefits as well as risks involved explained to him.

critical thinking

A synthesis of knowledge, experience, information gathered from patients and families Critical thinking attitudes Intellectual and professional standards

A nurse is evaluating an expected outcome for a patient that states heart rate will be less than 80 beats/min by 12/3. Which finding will alert the nurse that the goal has been met? A. Heart rate 78 beats/min on 12/3 B. Heart rate 78 beats/min on 12/4 C. Heart rate 80 beats/min on 12/3 D. Heart rate 80 beats/min on 12/4

A. Heart rate 78 beats/min on 12/3

2. Which statements are true regarding the elderly and pharmacokinetics? (Select all that apply.) 1. The levels of microsomal enzymes are decreased. 2. Fat content is increased because of decreased lean body mass. 3. Fat content is decreased because of increased lean body mass. 4. The number of intact nephrons is increased. 5. The number of intact nephrons is decreased. 6. Gastric pH is less acidic. 7. Gastric pH is more acidic.

ANS: 1, 2, 5, 6

A nurse is preparing to carry out interventions. Which resources will the nurse make sure are available? (Select all that apply.) A. Equipment B. Safe environment C. Confidence D. Assistive personnel E. Creativity

ANS: A, B, D

A nurse is implementing interventions for a group of patients. Which actions are nursing interventions? (Select all that apply.) A. Order chest x-ray for suspected arm fracture. B. Prescribe antibiotics for a wound infection. C. Reposition a patient who is on bed rest. D. Teach a patient preoperative exercises. E. Transfer a patient to another hospital unit.

ANS: C, D, E

1. A patient is to receive prednisone 7.5 mg PO daily. The tablets are available in a 2.5-mg strength. Identify how many tablets will the patient receive.

ANS:3 tablets 1 tablet:2.5 mg::x tablet:7.5 mg. (1 7.5) = (2.5 x); 7.5 = 2.5x; x = 3; therefore 7.5 mg = 3 tablets.

causes of spiritual distress

Acute illness Chronic illness Terminal illness Near-death experience

Health Belief Model

Addresses the relationship between a person's beliefs and behaviors

1. Which of the following is an example of a patient with a health disparity? (Select all that apply.) 1. A patient who has a homosexual sexual preference 2. A patient unable to access primary care services 3. A patient living with a chronic disease 4. A family who relies on public transportation 5. A patient who has had a history of smoking for 10 years

Answers: 2, 3, 5. Poor health status (chronic disease), disease risk factors (smoking history), poor health outcomes, and limited access to health care (unable to access primary care) are types of health disparities

Nursing Process

Assessment Diagnosis Planning Implementation Evaluation

A nurse is using professional standards to influence clinical decisions. What is the rationale for the nurse's actions? A. Establishes minimal passing standards for testing. B. Utilizes evidence-based practice based on nurses' needs. C. Bypasses the patient's feelings to promote ethical standards. D. Uses critical thinking for the highest level of quality nursing care.

D. Uses critical thinking for the highest level of quality nursing care.

Secondary level of prevention

Focuses on preventing the spread of disease, illness, or infection once it occurs. Activities are directed at diagnosis and prompt intervention.

knowledge life long process

Parental influence School-age: positively inflated Adolescence: can adversely affect self-concept and self-esteem Adulthood Self-concept affects perception of health

Identity stressors

Particularly vulnerable during adolescence Identity confusion

self esteem vs self concept

Self-esteem: how you feel Self-concept: how you think

illness

The way that individuals and families react to disease

Primary level of prevention

True prevention that lowers the chances that a disease will develop

oppression

a formal and informal system of advantages and disadvantages tied to our membership in social groups, such as those at work, at school, and in families

2. The patient reports to the nurse of being afraid to speak up regarding a desire to end care for fear of upsetting spouse and children. Which principle in the nursing code of ethics ensures that the nurse will promote the patient's cause? a. Advocacy b. Responsibility c. Confidentiality d. Accountability

a. Advocacy

10. A patient is undergoing major surgery and asks the nurse about a living will. He states, ―I don't want anybody else making decisions for me. And I don't want to prolong my life.‖ The patient is demonstrating which ethical term? a. Autonomy b. Beneficence c. Justice d. Veracity

a. Autonomy

13. The nurse will be injecting a drug into the superficial skin layers immediately underneath the epidermal layer of skin. Which route does this describe? a. Intradermal b. Subcutaneous c. Intramuscular d. Transdermal

a. Intradermal

During a follow-up visit, a female patient is describing new onset of marital discord with her terminally ill spouse to the hospice nurse. Which Kübler-Ross stage of dying is the patient experiencing? a. Denial b. Anger c. Bargaining d. Depression

b. Anger

5. The nurse is aware that confusion, forgetfulness, and increased risk for falls are common responses in an elderly patient who is taking which type of drug? a. Laxatives b. Anticoagulants c. Sedatives d. Antidepressants

c. Sedatives

The nurse teaches stress-reduction and relaxation training to a health education group of patients after cardiac bypass surgery. Which level of intervention is the nurse using? a. Primary b. Secondary c. Tertiary d. Quad

c. Tertiary

Family role and practices

the roles and organization of a family influence how each family member defines health and illness and values health practices

Advanced Practice Registered Nurse (APRN)

registered nurse educated at the master's or post-master's level in a specific role and for a specific population; examples include nurse practitioners, clinical nurse specialists, nurse anesthetists, and nurse midwives

Culture

remaining connected with their cultural heritage helping patients define their place in the world and express their spiriuality

Cultural Assessment

systematic and comprehensive examination of the cultural care values, beliefs, and practices of individuals, families, and communities

Nurse Educator

works primarily in schools of nursing, staff development departments of health care agencies, and patient education departments

self-concept

- an individuals view of self subjective - involves a complex mixture of unconscious and conscious thoughts, attitudes, and perceptions directly affects self-esteem

implementation

- collaborate with the other team members and patients to promote healthy self-concept - health promotion - acute care - restorative and continuing care

Core measures

- set of evidence-based, scientifically researched standards of care. - Key quality indicators that help health care institutions improve performance, accountability and costs. - intended to reduce health disparity

manager

-Establish an environment for collaborative patient-centered care to provide safe, quality, care with positive patient outcomes. -coordinates the activities of members of the nursing staff in delivering nursing care and has personal, policy, and budgetary responsibility for a specific nursing unit or agency.

World view of providers and patients

-Iceberg analogy tool -Conduct a comprehensive cultural assessment -These deeply held values reside "underneath the iceberg."

Spiritual well-being

-Meaning and purpose -Sense of peace or fulfillment -Interconnectedness between God or a higher power and others

Banner: From Novice to Expert

-Novice: Beginning nursing student or a nurse who has no previous level of experience -Advanced Beginner: A nurse that has some level of experience with the situation. The nurse is able to identify meaningful aspects or principles of nursing care. -Competent: A nurse who has been in the same clinical position for 2-3 years. The nurse understands the organization and specific care required by the type of patients -Proficient: A nurse with more than 2-3 years of experience in the same clinical position. This nurse perceives a patients clinical situation as a whole, is able to assess an entire situation, and can readily transfer knowledge gained from multiple previous experiences to a situation. This nurse focuses on managing care as opposed to managing and performing skills. -Expert: A Nurse with a diverse experience who has an intuitive grasp of a existing or potential clinical problem. This nurse is able to zero in on a problem and focus on multiple dimensions of a situation. They are skilled at identifying both patient centered problems and problems related to the healthcare system or perhaps the needs of the novice nurse.

Constructs of spirituality

-Self transcendence -Connectedness -Faith -Hope -Inner strength and peace -Meaning and purpose in life

Health literacy assessment tools

-Short assessment of health literacy -The rapid estimate of adult literacy in medicine

Inner strength and peace

-Source of energy, helps people stay open to change & life challenges -Fosters calm, positive and peaceful feelings despite chaos, fear, uncertainty

career development

-majority of nurses practice in hospital settings, followed by community based care, ambulatory care, home care, and nursing homes/extended care settings. -nursing provides an opportunity for you to commit to lifelong learning and career development. -your career path is limitless -These career opportunities include advanced practice registered nurses (APRNs), nurse researchers, nurse risk managers, quality improvement nurses, consultants and entrepreneurs.

A nurse is prioritizing care. Match the level of priority to the patients. A. Patient that needs to be turned to prevent pneumonia B. Patient with acute asthma attack C. Patient who will be discharged in 2 days who needs teaching 1. High priority 2. Intermediate priority 3. Low priority

1) ANS: B 2) ANS: A 3) ANS: C

Nursing Process

1) Assessment: The RN collects all pertinent data and information, related to the healthcare consumers health or the situation 2) Diagnosis: The RN analyzes the assessment data to determine the actual or potential diagnoses, problems and issues. 3) Outcomes Identification: The RN identifies expected outcomes for a plan individualized to the healthcare consumer or the situation. 4) Planning: The RN develops a plan that prescribes strategies to attain expected, measurable outcomes. 5) Implementation: The RN implements the identified plan. a) Coordination of care: The RN coordinated care delivery. b) Health Teaching and Health Promotion: The RN employs strategies to promote health and a safe environment. 6) Evaluation: The RN evaluates progress toward attainment of outcomes.

ANA Standards of Professional Performance

1) Ethics: The RN practices ethically 2) Culturally Congruent Care: The RN practices in a manner that is congruent(in agreement or harmony with) with cultural diversity and inclusion principles. 3) Communication: The RN communicates effectively in all areas of practice. 4) Collaboration: The RN collaborates with health care consumer and other key stakeholders in the conduct of nursing practice. 5) Leadership: The RN demonstrates leadership in the professional practice setting and the profession. 6) Education: The RN seeks knowledge and competency that reflects current nursing practice and promotes futuristic thinking. 7) Evidence-Based Practice and Research: The RN integrates evidence and research findings into practice. 8) Quality of Practice: The RN contributes to quality nursing practice. 9) Professional Practice Evaluation: The RN evaluates one's own and others nursing practice. 10)Resources Utilization: The RN uses appropriate resources to plan and provide and sustain evidence-based nursing services that are safe, effective and fiscally responsible. 11) Environmental Health: The RN practices in an environmentally safe and healthy manner.

4. A nurse is planning care for an older-adult patient who is experiencing pain. Which statement made by the nurse indicates the supervising nurse needs to follow up? 1. "As adults age, their ability to perceive pain decreases." 2. "Older patients may have low serum albumin in their blood, causing toxic effects of analgesic drugs." 3. "Patients who have dementia probably experience pain, and their pain is not always well controlled." 4. "It is safe to administer opioids to older adults as long as you start with small doses and frequently assess the patient's response to the medication."

1. "As adults age, their ability to perceive pain decreases."

3. A nurse teaches the patient about the gate control theory. Which statement made by a patient reflects a correct understanding about the relationship between the gate control theory of pain and the use of meditation to relieve pain? 1. "Meditation controls pain by blocking pain impulses from coming through the gate." 2. "Meditation alters the chemical composition of pain neuroregulators, which closes the gate." 3. "Meditation will help me sleep through the pain because it opens the gate." 4. "Meditation stops the occurrence of pain stimuli."

1. "Meditation controls pain by blocking pain impulses from coming through the gate."

15. A nurse hears a co-worker state that anybody could be a nurse since it is so automated with infusion devices and electronic monitoring; technology is doing the work. What is the nurse's best response? 1. "Technology use has to be combined with nursing judgment." 2. "The focus of effective nursing care is technology." 3. "If it's so easy, why don't you do it?" 4. "That is true in the twentieth century."

1. "Technology use has to be combined with nursing judgment."

26. The nurse is teaching a student nurse about pain assessment scales. Which statement by the student indicates effective teaching? 1. "You cannot use a pain scale to compare the pain of my patient with the pain of your patient." 2. "When patients say they don't need pain medication, they aren't in pain." 3. "A patient's behavior is more reliable than the patient's report of pain." 4. "Pain assessment scales determine the quality of a patient's pain."

1. "You cannot use a pain scale to compare the pain of my patient with the pain of your patient."

12. A nurse is providing discharge teaching for a patient with a fractured humerus. The patient is going home with a prescription for hydrocodone. Which important patient education should the nurse provide? 1. "You need to drink plenty of fluids and eat a diet high in fiber." 2. "Narcotics can be addictive, so do not take them unless you are in severe pain." 3. "Be sure to eat a meal high in fat before taking the medication, to avoid a stomach ulcer." 4. "As your pain severity lessens, you will begin to give yourself once-daily intramuscular injections."

1. "You need to drink plenty of fluids and eat a diet high in fiber."

16. A nurse in a long-term care setting that is funded by Medicare and Medicaid is completing standardized protocols for assessment and care planning for reimbursement. Which task is the nurse completing? 1. A minimum data set 2. An admission assessment and acuity level 3. A focused assessment/specific body system 4. An intake assessment form and auditing phase

1. A minimum data set

23. A nurse is caring for a group of patients. Which patient will the nurse see first to best manage patient needs? 1. A patient who received morphine and has a pulse of 62 beats/min, respirations 10 breaths/min, and blood pressure 110/60 mm Hg. 2. A patient lying very still in bed who reports no pain but is pale with warm, dry skin. 3. A patient with severe pain who is nauseated and feels like he or she is about to vomit. 4. A patient writhing and moaning from abdominal pain after abdominal surgery

1. A patient who received morphine and has a pulse of 62 beats/min, respirations 10

1. During the development of a new drug, which would be included in the study by the researcher to prevent any bias or unrealistic expectations of the new drug's usefulness? 1. A placebo 2. FDA approval 3. Informed consent 4. Safety information

1. A placebo

2. A nurse exchanges information with the oncoming nurse about a patient's care. Which action did the nurse complete? 1. A verbal report 2. An electronic record entry 3. A referral 4. An acuity rating

1. A verbal report

31. The nurse is caring for a group of patients. Which task may the nurse delegate to the unlicensed assistive personnel (UAP)? 1. Administer a back massage to a patient with pain. 2. Assessment of pain for a patient reporting abdominal pain. 3. Administer patient-controlled analgesia for a postoperative patient. 4. Assessment of vital signs in a patient receiving epidural analgesia.

1. Administer a back massage to a patient with pain.

The nurse is caring for a patient who has just had a near-death experience (NDE) following a cardiac arrest. Which intervention by the nurse best promotes the spiritual well-being of the patient after the NDE? 1. Allowing the patient to discuss the experience 2. Referring the patient to pastoral care 3. Having the patient talk to another patient who had an NDE 4. Offering to pray for the patient

1. Allowing the patient to discuss the experience

Professional responsibilities and roles

1. Autonomy and Accountability 2. Caregiver 3. Advocate 4. Educator 5. Communicator 6. Manager

1. The patient is receiving two different drugs. At current dosages and dosage forms, both drugs have the same concentration of the active ingredient. Which term is used to identify this principle? 1. Bioequivalent 2. Synergistic 3. Prodrugs 4. Steady state

1. Bioequivalent

7. Which action can the nurse take legally when charting on a patient's record? 1. Charts in a legible manner. 2. States the patient is belligerent. 3. Writes entry for another nurse. 4. Uses correction fluid to correct error.

1. Charts in a legible manner.

1. The nurse is developing a human needs statement for a patient who has a new diagnosis of heart failure. Identification of human needs statements occur with which of these activities? 1. Collection of patient data 2. Administering interventions 3. Deciding on patient outcomes 4. Documenting the patient's behavior

1. Collection of patient data

11. A nurse is teaching about the primary focus of community wellness. Which information should the nurse include in the teaching session? 1. Coordination of health care services 2. Effective cost containment for services 3. Appropriate service delivery to service population 4. Identification of services needed to address individual needs

1. Coordination of health care services

3. A nurse is auditing and monitoring patients' health records. Which action is the nurse taking? 1. Determining the degree to which standards of care are met by reviewing patients' health records 2. Realizing that care not documented in patients' health records still qualifies as care provided 3. Basing reimbursement upon the diagnosis-related groups documented in patients' records 4. Comparing data in patients' records to determine whether a new treatment had better outcomes than the standard treatment

1. Determining the degree to which standards of care are met by reviewing patients' health records

7. When reviewing the various schedules of controlled drugs, the nurse knows that which description correctly describes Schedule II drugs? 1. Drugs with high potential for abuse that have accepted medical use 2. Drugs with high potential for abuse that do not have accepted medical use 3. Medically accepted drugs that may cause moderate physical or psychologic dependence 4. Medically accepted drugs with limited potential for causing physical or psychologic dependence

1. Drugs with high potential for abuse that have accepted medical use

4. A nurse is working in a health care organization that has achieved Magnet status. Which components are indicators of this status? (Select all that apply.) 1. Empirical quality results 2. Structural empowerment 3. Transformational leadership 4. Exemplary professional practice 5. Willingness to recommend the agency

1. Empirical quality results 2. Structural empowerment 3. Transformational leadership 4. Exemplary professional practice

32. A nurse is caring for a patient with chronic pain from arthritis. Which action is best for the nurse to take? 1. Give pain medications around the clock. 2. Administer pain medication before any activity. 3. Give pain medication after the pain is a 7/10 on the pain scale. 4. Administer pain medication only when nonpharmacological measures have failed.

1. Give pain medications around the clock

11. The nurse is trying to give a liquid medication to a 2 1/2-year-old child and notes that the medication has a strong taste. Which technique is the best way for the nurse to give the medication to this child? 1. Give the medication with a spoonful of ice cream. 2. Add the medication to the child's bottle. 3. Tell the child you have candy for him. 4. Add the medication to a cup of milk.

1. Give the medication with a spoonful of ice cream.

18. A nurse is completing an Outcome and Assessment Information Set (OASIS) data set on a patient. The nurse works in which area of patient care? 1. Home health 2. Intensive care unit 3. Skilled nursing facility 4. Long-term care facility

1. Home health

27. A nurse is teaching the staff about health care reimbursement. Which information should the nurse include in the teaching session? 1. Home health, long-term care, and hospital nurses' documentation can affect reimbursement for health care. 2. A clinical information system must be installed by 2014 to obtain health care reimbursement. 3. A "near miss" helps determine reimbursement issues for health care. 4. HIPAA is the basis for establishing reimbursement for health care.

1. Home health, long-term care, and hospital nurses' documentation can affect reimbursement for health care.

Erikson's Theory

1. Infancy (Birth to 18 Months) 2. Early Childhood (2-3 years) 3. Preschool (3-5 years) 4. School Age (6-11 Years) 5. Adolescence (12-18 years) 6. Young Adulthood (19-40 Years) 7. Middle Adulthood (41-65 Years) 8 Maturity (65 Years-Death)

5. The nurse is caring for two patients; both are having a hysterectomy. The first patient is having the hysterectomy after a complicated birth. The second patient has uterine cancer. What will most likely influence the experience of pain for these two patients? 1. Meaning of pain 2. Neurological factors 3. Competency of the surgeon 4. Postoperative support personnel

1. Meaning of pain

16. A nurse is completing a minimum data set. Which area is the nurse working? 1. Nursing center 2. Psychiatric facility 3. Rehabilitation center 4. Adult day care center

1. Nursing center

6. For accurate medication administration to pediatric patients, the nurse must consider which of these factors? 1. Organ maturity 2. Renal output 3. Body temperature 4. Height

1. Organ maturity

11. A patient who has advanced cancer is receiving opioid medications around the clock to keep him comfortable as he nears the end of his life. Which term best describes this type of therapy? 1. Palliative therapy 2. Maintenance therapy 3. Empiric therapy 4. Supplemental therapy

1. Palliative therapy

3. The nurse is administering parenteral drugs. Which statement is true regarding parenteral drugs? 1. Parenteral drugs bypass the first-pass effect. 2. Absorption of parenteral drugs is affected by reduced blood flow to the stomach. 3. Absorption of parenteral drugs is faster when the stomach is empty. 4. Parenteral drugs exert their effects while circulating in the bloodstream.

1. Parenteral drugs bypass the first-pass effect.

29. A nurse is caring for a patient with rheumatoid arthritis who is now going to be taking 2 acetaminophen tablets every 6 hours to control pain. Which part of the patient's social history is the nurse most concerned about? 1. Patient drinks 1 to 2 glasses of wine every night. 2. Patient smokes 2 packs of cigarettes a day. 3. Patient occasionally uses marijuana. 4. Patient takes antianxiety medications.

1. Patient drinks 1 to 2 glasses of wine every night.

1. The nurse is administering ibuprofen to an older patient. Which assessment data causes the nurse to hold the medication? (Select all that apply.) 1. Patient states allergy to aspirin. 2. Patient states joint pain is 2/10 and intermittent. 3. Patient reports past medical history of gastric ulcer. 4. Patient reports last bowel movement was 4 days ago. 5. Patient experiences respiratory depression after administration of an opioid medication.

1. Patient states allergy to aspirin. 3. Patient reports past medical history of gastric ulcer.

12. A nurse is using research findings to improve clinical practice. Which technique is the nurse using? 1. Performance scores 2. Integrated delivery networks 3. Nursing-sensitive outcomes 4. Utilization review committees

1. Performance scores

1. Which government-instituted programs should the nurse include in a teaching session about controlling health care costs? (Select all that apply.) 1. Professional standards review organizations 2. Prospective payment systems 3. Diagnosis-related groups 4. Third-party payers 5. "Never events"

1. Professional standards review organizations 2. Prospective payment systems 3. Diagnosis-related groups

13. Which finding indicates the best quality improvement process? 1. Staff identifies the wait time in the emergency department is too long. 2. Administration identifies the design of the facility's lobby increases patient stress. 3. Director of the hospital identifies the payment schedule does not pay enough for overtime. 4. Health care providers identify the inconsistencies of some of the facility's policy and procedures.

1. Staff identifies the wait time in the emergency department is too long.

1. The nurse is reviewing the teaching plan for a clinic patient who was seen for a sinus infection. Which of these outcomes reflect the affective domain of learning? 1. The patient will take the prescribed antibiotic for the full 14 days of the prescription. 2. The patient will demonstrate correct nasal spray self-administration. 3. The patient will list signs and symptoms that need to be reported immediately if they occur. 4. The patient will list measures to take to reduce allergy triggers at home.

1. The patient will take the prescribed antibiotic for the full 14 days of the prescription

12. A nurse needs to begin discharge planning for a patient admitted with pneumonia and a congested cough. When is the best time the nurse should start discharge planning for this patient? 1. Upon admission 2. Right before discharge 3. After the congestion is treated 4. When the primary care provider writes the order

1. Upon admission

2. A nurse is teaching the staff about the Institute of Medicine competencies. Which examples indicate the staff has a correct understanding of the teaching? (Select all that apply.) 1. Use informatics. 2. Use transparency. 3. Apply globalization. 4. Apply quality improvement. 5. Use evidence-based practice.

1. Use informatics 4. Apply quality improvement. 5. Use evidence-based practice.

22. The nurse is caring for a 4-year-old child who is demonstrating signs of pain. Which technique will the nurse use to best assess pain in this child? 1. Use the FACES scale. 2. Check to see what previous nurses have charted. 3. Ask the parents if they think their child is in pain. 4. Have the child rate the level of pain on a 0 to 10 pain scale.

1. Use the FACES scale.

26. A patient has a diagnosis of pneumonia. Which entry should the nurse chart to help with financial reimbursement? 1. Used incentive spirometer to encourage coughing and deep breathing. Lung congested upon auscultation in lower lobes bilaterally. Pulse oximetry 86%. Oxygen per nasal cannula applied at 2 L/min per standing order. 2. Cooperative, patient coughed and deep breathed using a pillow as a splint. Stated, "felt better." Finally, patient had no complaints. 3. Breathing without difficulty. Sitting up in bed watching TV. Had a good day. 4. Status unchanged. Remains stable with no abnormal findings. Checked every 2 hours.

1. Used incentive spirometer to encourage coughing and deep breathing. Lung congested upon auscultation in lower lobes bilaterally. Pulse oximetry 86%. Oxygen per nasal cannula applied at 2 L/min per standing order.

three outcomes for patient to achieve harmony

1. expresses an acceptance to illness 2. reports the ability to rely on family for support 3. initates social interactions with fam/friends

Cultural Assessment guide

1. health beliefs and practices 2. faith-based influences and special rituals 3. language and communication 4. parenting styles and family roles 5. sources of support beyond the family 6. dietary pratices

Factors influencing self-concept

1. identity stressors 2. body image stressors 3. role performance 4.self-esteem stressors

2. The patient is to receive oral guaifenesin twice a day. Today, the nurse was busy and gave the medication 2 hours after the scheduled dose was due. What type of problem does this represent? 1. ―Right time‖ 2. ―Right dose‖ 3. ―Right route‖ 4. ―Right medication‖

1. ―Right time‖

The nurse is assessing the characteristics of a patient's pain. Match the characteristic to the question a nurse will ask to determine that specific characteristic. 1. Could you rate your pain on a scale of 0 to 10? 2. How often does it recur? 3. Could you point to the area of pain? 4. Do certain activities worsen the pain? 5. What does the pain feel like? A. Timing B. Location C. Severity D. Quality E. Aggravating factors

1.B 2.C 3.A 4.E 5.D

24. A nurse is caring for a patient diagnosed with chronic pain. Which statement by the nurse indicates an understanding of pain management? 1. "This patient says the pain is a 5 but is not acting like it. I am not going to give any pain medication." 2. "I need to reassess the patient's pain 1 hour after administering oral pain medication." 3. "It wasn't time for the patient's medication, so when it was requested, I gave a placebo." 4. "The patient is sleeping, so I pushed the PCA button."

2. "I need to reassess the patient's pain 1 hour after administering oral pain medication."

25. The nurse is assessing how a patient's pain is affecting mobility. Which assessment question is most appropriate? 1. "Have you considered working with a physical therapist?" 2. "What activities, if any, has your pain prevented you from doing?" 3. "Would you please rate your pain on a scale from 0 to 10 for me?" 4. "When does your pain medication typically take effect on your pain?"

2. "What activities, if any, has your pain prevented you from doing?"

1. The nurse is reviewing medication errors. Which situation is an example of a medication error? 1. A patient refuses her morning medications. 2. A patient receives a double dose of a medication because the nurse did not cut the pill in half. 3. A patient develops hives after having started an IV antibiotic 24 hours earlier. 4. A patient complains of severe pain still present 60 minutes after a pain medication was given.

2. A patient receives a double dose of a medication

1. A patient has recently had surgery. Which action is best for the nurse to take to assess this patient's pain? 1. Assess the patient's body language. 2. Ask the patient to rate the level of pain. 3. Observe the cardiac monitor for increased heart rate. 4. Have the patient describe the effect of pain on the ability to cope.

2. Ask the patient to rate the level of pain.

13. A patient is being discharged home. Which information should the nurse include? 1. Acuity level 2. Community resources 3. Standardized care plan 4. Signature for verbal order

2. Community resources

4. During a period of time when the computerized medication order system was down, the prescriber wrote admission orders, and the nurse is transcribing them. The nurse is having difficulty transcribing one order because of the prescriber's handwriting. Which is the best action for the nurse to take at this time? . 1. Ask a colleague what the order says. 2. Contact the prescriber to clarify the order. 3. Wait until the prescriber makes rounds again to clarify the order. 4. Ask the patient what medications he takes at home.

2. Contact the prescriber to clarify the order.

9. A nurse has instructed the patient regarding the proper use of crutches. The patient went up and down the stairs using crutches with no difficulties. Which information will the nurse use for the "I" in PIE charting? 1. Patient went up and down stairs 2. Demonstrated use of crutches 3. Used crutches with no difficulties 4. Deficient knowledge related to never using crutches

2. Demonstrated use of crutches

3. A nurse is evaluating care based upon the nursing quality indicators. Which areas should the nurse evaluate? (Select all that apply.) 1. Patient satisfaction level 2. Hospital readmission rates 3. Nursing hours per patient day 4. Patient falls/falls with injuries 5. Value stream analysis for quality

2. Hospital readmission rates 3. Nursing hours per patient day 4. Patient falls/falls with injuries

7. A patient is receiving opioid medication through an epidural infusion. Which action will the nurse take to protect the patient's safety? 1. Restrict fluid intake. 2. Label the tubing that leads to the epidural catheter. 3. Apply a gauze dressing to the epidural catheter insertion site. 4. Ask the nursing assistive personnel to check on the patient at least once every 2 hours.

2. Label the tubing that leads to the epidural catheter.

12. During a busy night shift, a new nurse administered an unfamiliar medication without checking it in a drug handbook. Later that day, the patient had a severe reaction because he has renal problems, which was a contraindication to that drug. The nurse may be liable for which of these? 1. Medical negligence 2. Nursing negligence 3. Nonmaleficence 4. Autonomy

2. Nursing negligence

20. Which action will the nurse take when taking a telephone order? 1. Print out a copy of the order once entered into the electronic health record. 2. Read back the order as written to the health care provider for verification. 3. Ask that another registered nurse listen to the call over an extension line. 4. Verify that the health care provider will write the order within 24 hours.

2. Read back the order as written to the health care provider for verification.

28. A nurse is discussing the advantages of a nursing clinical information system. Which advantage should the nurse describe? 1. Varied clinical databases 2. Reduced errors of omission 3. Increased hospital costs 4. More time to read charts

2. Reduced errors of omission

A patient has just learned she has been diagnosed with a malignant brain tumor. She is alone; her family will not be arriving from out of town for an hour. The nurse has been caring for her for only 2 hours but has a good relationship with her. What is the most appropriate intervention for support of her spiritual well-being at this time? 1. Make a referral to a professional spiritual care adviser. 2. Sit down and talk with the patient; have her discuss her feelings and listen attentively 3. Move the patient's Bible from her bedside cabinet drawer to the top of the over-bed table. 4. Ask the patient whether she would like to lea

2. Sit down and talk with the patient; have her discuss her feelings and listen attentively Establishing presence contributes to a patient's sense of well-being. It helps to prevent emotional and environmental isolation. Automatically making a referral to a spiritual care

13. A patient arrives at the emergency department experiencing a headache and rates the pain as 7 on a 0 to 10 pain scale. Which nonpharmacological intervention does the nurse implement for this patient while awaiting orders for pain medication from the health care provider? 1. Reassures the patient that the provider will come to the emergency department soon. 2. Softly plays music that the patient finds relaxing. 3. Frequently reassesses the patient's pain scores. 4. Teaches the patient how to do yoga.

2. Softly plays music that the patient finds relaxing

8. The nurse is trying to determine risk factors unique to home care patients. What resource should the nurse access? 1. Pew Health Professions Commission 2. The Outcome and Assessment Information Set (OASIS) 3. American Nurses Credentialing Center (ANCC) Magnet Recognition Program 4. Hospital Consumer of Assessment of Healthcare Providers and Systems (HCAHPS)

2. The Outcome and Assessment Information Set (OASIS)

28. The nurse is assessing a patient for opioid tolerance. Which finding supports the nurse's assessment? 1. The patient needed a substantial dose of naloxone. 2. The patient needs increasingly higher doses of opioid to control pain. 3. The patient no longer experiences sedation from the usual dose of opioid. 4. The patient asks for pain medication close to the time it is due around the clock.

2. The patient needs increasingly higher doses of opioid to control pain.

16. A patient injured in a motor vehicle crash 2 days ago is experiencing pain and is receiving patient-controlled analgesia (PCA). Which assessment finding indicates effective pain management with the PCA? 1. The patient is sleeping and is difficult to arouse. 2. The patient rates pain at a level of 2 on a 0 to 10 scale. 3. The patient has sufficient medication left in the PCA syringe. 4. The patient presses the control button to deliver pain medication.

2. The patient rates pain at a level of 2 on a 0 to 10 scale.

11. A nurse is caring for a patient who recently had spinal surgery. The nurse knows that patients usually experience acute pain following this type of surgery. The patient refuses to get up and walk and is not moving around in the bed. However, the patient is stoic and denies experiencing pain at this time. What most likely explains this patient's behavior? 1. surgery successfully cured the patient's pain. 2. The patient's culture is possibly influencing the patient's experience of pain. 3. The The primary health care provider did not prescribe the correct amount of medication. 4. The nurse is allowing personal beliefs about pain to influence pain management at this time.

2. The patient's culture is possibly influencing the patient's experience of pain.

1. The nurse can prevent medication errors by following which principles? (Select all that apply.) 1. Assess for allergies after giving medications. 2. Use two patient identifiers before giving medications. 3. Always following the rights of medication administration. 4. Minimize the use of verbal and telephone orders. 5. Use trade names instead of generic names to avoid confusion.

2. Use two patient identifiers before giving medications. 3. Always following the rights of medication administration. 4. Minimize the use of verbal and telephone orders.

3. During a nursing assessment, which question by the nurse allows for greater clarification and additional discussion with the patient? 1. ―Are you allergic to iodine? 2. ―What type of reaction did you have to penicillin? 3. ―Have you had a reaction to this drug? 4. ―Are you taking this medication with meals?

2. ―What type of reaction did you have to penicillin?

1. Which drugs would be affected by the first-pass effect when administered? (Select all that apply.) 1. Morphine given by IV push injection 2. Sublingual nitroglycerin tablet 3. Diphenhydramine elixir 4. Levothyroxine (Synthroid) tablet 5. Transdermal nicotine patches 6. Esomeprazole capsule 7. Penicillin given by IV piggyback infusion

3, 4, 6

10. A nurse is caring for a patient who is experiencing pain following abdominal surgery. Which information is important for the nurse to share with the patient when providing patient education about effective pain management? 1. "To prevent overdose, you need to wait to ask for pain medication until you begin to experience pain." 2. "You should take your medication after you walk to make sure you do not fall while you are walking." 3. "We should work together to create a schedule to provide regular dosing of medication." 4. "When you experience severe pain, you will need to take oral pain medications."

3. "We should work together to create a schedule to provide regular dosing of medication."

2. A nurse is caring for a patient who recently had abdominal surgery and is experiencing severe pain. The patient's blood pressure is 110/60 mm Hg, and heart rate is 60 beats/min. Additionally, the patient does not appear to be in any physical distress. Which response by the nurse is most therapeutic? 1. "Your vitals do not show that you are having pain; can you describe your pain?" 2. "OK, I will go get you some narcotic pain relievers immediately." 3. "What would you like to try to alleviate your pain?" 4. "You do not look like you are in pain."

3. "What would you like to try to alleviate your pain?"

20. The nurse has brought a patient the scheduled pain medication. The patient asks the nurse to wait to give pain medication until the time for the dressing change, which is 2 hours away. Which response by the nurse is most therapeutic? 1. "This medication will still be providing you relief at the time of your dressing change." 2. "OK, swallow this pain pill, and I will return in a minute to change your dressing." 3. "Would you like medication to be given for dressing changes in addition to your regularly scheduled medication?" 4. "Your medication is scheduled for this time, and I can't adjust the time for you. 5. I'm sorry, but you must take your pill right now."

3. "Would you like medication to be given for dressing changes in addition to your regularly scheduled medication?"

21. A nurse obtained a telephone order from a primary care provider for a patient in pain. Which chart entry should the nurse document? 1. 12/16/20XX 0915 Morphine, 2 mg, IV every 4 hours for incisional pain. VO Dr. Day/J. Winds, RN, read back. 2. 12/16/20XX 0915 Morphine, 2 mg, IV every 4 hours for incisional pain. TO J. Winds, RN, read back. 3. 12/16/20XX 0915 Morphine, 2 mg, IV every 4 hours for incisional pain. TO Dr. Day/J. Winds, RN, read back. 12/16/20XX 0915 Morphine, 2 mg, IV every 4 hours for incisional pain. TO J. Winds, RN

3. 12/16/20XX 0915 Morphine, 2 mg, IV every 4 hours for incisional pain. TO Dr. Day/J. Winds, RN, read back.

6. The nurse is preparing pain medications. To which patient does the nurse anticipate administering an opioid fentanyl patch? 1. A 15-year-old adolescent with a fractured femur 2. A 30-year-old adult with cellulitis 3. A 50-year-old patient with prostate cancer 4. An 80-year-old patient with a broken hip

3. A 50-year-old patient with prostate cancer

12. The patient is stating that he has a headache and asks the nurse which over-the-counter medication form would work the fastest to help reduce the pain. Which medication form will the nurse suggest? 1. A capsule 2. A tablet 3. A powder 4. An enteric-coated tablet

3. A powder

7. During an admission assessment, the nurse discovers that the patient does not speak English. Which is considered the ideal resource for translation? 1. A family member of the patient 2. A close family friend of the patient 3. A translator who does not know the patient 4. Prewritten note cards with both English and the patient's language

3. A translator who does not know the patient

3. A nurse is teaching a family about health care plans. Which information from the nurse indicates a correct understanding of the Affordable Care Act? 1. A family can choose whether to have health insurance with no consequences. 2. Primary care physician payments from Medicaid services can equal Medicare. 3. Adult children up to age 26 are allowed coverage on the parent's plan. 4. Quality hospital outcome scores are tied directly to patient satisfaction.

3. Adult children up to age 26 are allowed coverage on the parent's plan.

21. A nurse receives an order from a health care provider to administer hydrocodone and acetaminophen to a patient who is experiencing 8/10 postsurgical pain. The order is to give 2 tablets every 6 hours by mouth as needed for pain. What is the nurse's next best action? 1. Give the medication to the patient immediately because the patient is experiencing severe pain. 2. Ask the health care provider for a nonsteroidal antiinflammatory drug (NSAID) order. 3. Ask the health care provider to verify the dosage and frequency of the medication. 4. Give the medication in addition to playing soothing music for the patient.

3. Ask the health care provider to verify the dosage and frequency of the medication.

6. The medication order reads, ―Give ondansetron 4 mg, 30 minutes before beginning chemotherapy to prevent nausea.‖ The nurse notes that the route is missing from the order. What is the nurse's best action? 1. Give the medication intravenously because the patient might vomit. 2. Give the medication orally because the tablets are available in 4-mg doses. 3. Contact the prescriber to clarify the route of the medication ordered. 4. Hold the medication until the prescriber returns to make rounds.

3. Contact the prescriber to clarify the route of the medication ordered.

23. A hospital is using a computer system that allows all health care providers to use a protocol system to document the care they provide. Which type of system/design will the nurse be using? 1. Clinical decision support system 2. Nursing process design 3. Critical pathway design 4. Computerized provider order entry system

3. Critical pathway design

1. The nurse is caring for a patient whose insurance coverage is Medicare. The nurse should consider which information when planning care for this patient? 1. Capitation provides the hospital with a means of recovering variable charges. 2. The hospital will be paid for the full cost of the patient's hospitalization. 3. Diagnosis-related groups (DRGs) provide a fixed reimbursement of cost. 4. Medicare will pay the national average for the patient's condition

3. Diagnosis-related groups (DRGs) provide a fixed reimbursement of cost.

11. A nurse is a member of an interdisciplinary team that uses critical pathways. According to the critical pathway, on day 2 of the hospital stay, the patient should be sitting in the chair. It is day 3, and the patient cannot sit in the chair. What should the nurse do? 1. Add this data to the problem list. 2. Focus chart using the DAR format. 3. Document the variance in the patient's record. 4. Report a positive variance in the next interdisciplinary team meeting.

3. Document the variance in the patient's record.

1. Drug transfer to the fetus is more likely during the last trimester of pregnancy for which reason? 1. Decreased fetal surface area 2. Increased placental surface area 3. Enhanced blood flow to the fetus 4. Increased amount of protein-bound drug in maternal circulation

3. Enhanced blood flow to the fetus

A 30-year-old patient diagnosed with major depressive disorder has a nursing diagnosis of Situational Low Self-Esteem related to negative view of self. Which of the following are appropriate interventions by the nurse? (Select all that apply.) 1. Encourage reconnecting with high school friends. 2. Role-play to increase assertiveness skills. 3. Focus on identifying strengths and accomplishments. 4. Provide time for journaling to explore underlying thoughts and feelings. 5. Explore new job opportunities.

3. Focus on identifying strengths and accomplishments. 4. Provide time for journaling to explore underlying thoughts and feelings. Focusing on strengths and accomplishments to minimize the emphasis on failures helps the patient alter distorted and negative thinking. Journaling can allow a patient to explore thoughts and feelings that can promote insight and eventual behavioral change. The other interventions represent the nurse imposing ideas on what needs to occur for the patient to be healthier; allowing the patient to direct the change process is important.

22. A nurse is teaching the staff about informatics. Which information from the staff indicates the nurse needs to follow up? 1. To be proficient in informatics, a nurse should be able to discover, retrieve, and use information in practice. 2. A nurse needs to know how to find, evaluate, and use information effectively. 3. If a nurse has computer competency, the nurse is competent in informatics. 4. Nursing informatics is a recognized specialty area of nursing practice.

3. If a nurse has computer competency, the nurse is competent in informatics.

9. The patient is experiencing chest pain and needs to take a buccal form of nitroglycerin. Where does the nurse instruct the patient to place the tablet? 1. Under the tongue 2. On top of the tongue 2. At the back of the throat 3. In the space between the cheek and the gum

3. In the space between the cheek and the gum

5. A nurse has provided care to a patient. Which entry should the nurse document in the patient's record? 1. Status unchanged, doing well. 2. Patient seems to be in pain and states, "I feel uncomfortable." 3. Left knee incision 1 inch in length without redness, drainage, or edema. 4. Patient is hard to care for and refuses all treatments and medications. Family is present

3. Left knee incision 1 inch in length without redness, drainage, or edema.

2. The nurse is reviewing a list of verbal medication orders. Which is the proper notation of the dose of the drug ordered? 1. Levothyroxine .75 mg 2. Levothyroxine .750 mg 3. Levothyroxine 0.75 mg 4. Levothyroxine 0.750 mg

3. Levothyroxine 0.75 mg

5. For which cultural group must the health care provider respect the value placed on preserving harmony with nature and the belief that disease is a result of ill spirits? 1. Hispanics 2. Asian Americans 3. Native Americans 4. African Americans

3. Native Americans

12. The nurse is preparing to give an injection to a 4-year-old child. Which intervention is age appropriate for this child? 1. Give the injection without any advanced preparation. 2. Give the injection, and then explain the reason for the procedure afterward. 3. Offer a brief, concrete explanation of the procedure at the patient's level and with the parent or caregiver present. 4. Prepare the child in advance with details about the procedure without the parent or caregiver present.

3. Offer a brief, concrete explanation of the procedure at the patient's level and with the parent or caregiver present.

5. When taking a telephone order for a medication, which action by the nurse is most appropriate? 1. Verify the order with the charge nurse. 2. Call back the prescriber to review the order. 3. Repeat the order to the prescriber before hanging up the telephone. 4. Ask the pharmacist to double-check the order.

3. Repeat the order to the prescriber before hanging up the telephone.

6. During morning medication administration, the nurse discovered an error on the electronic MAR before the medication was given. Which action by the nurse is appropriate for this near-miss? 1. Correct the MAR error but say nothing because nothing happened. 2. Notify the pharmacy about the error they almost caused. 3. Report the near-miss using the facility's recommended protocol, and correct the error on the MAR. 4. Report the near-miss to the next shift before the next dose is due.

3. Report the near-miss using the facility's recommended protocol, and correct the error on the MAR.

5. The nurse is applying for a position with a home care organization that specializes in spinal cord injury. In which type of health care facility does the nurse want to work? 1. Secondary acute 2. Continuing 3. Restorative 4. Tertiary

3. Restorative

8. The nurse is administering drugs to neonates and will consider which factor may contribute the most to drug toxicity? 1. The lungs are immature. 2. The kidneys are small. 3. The liver is not fully developed. 4. Excretion of the drug occurs quickly

3. The liver is not fully developed.

3. During discharge patient teaching, the nurse reviews prescriptions with a patient. Which statement is correct about refills for an analgesic that is classified as Schedule C-III? 1. No prescription refills are permitted. 2. Refills are allowed only by written prescription. 3. The patient may have no more than five refills in a 6-month period. 4. Written prescriptions expire in 12 months.

3. The patient may have no more than five refills in a 6-month period.

17. A nurse is charting. Which event is critical for the nurse to document? 1. The patient had a good day with no complaints. 2. The family is demanding and argumentative. 3. The patient received a pain medication. 4. The family is poor and had to go on welfare.

3. The patient received a pain medication.

8. The nurse is performing an assessment of a newly admitted patient. Which is an example of subjective data? 1. Weight 155 pounds 2. Pulse 72 beats/minute 3. The patient reports that he uses the herbal product ginkgo 4. The patient's complete blood count results

3. The patient reports that he uses the herbal product ginkgo

2. The nurse is developing a care plan for a patient who will be self-administering a metered-dose inhaler. Which statement reflects a measurable outcome? 1. The patient will know about self-administration of a metered-dose inhaler. 2. The patient will understand the principles of self-administration of a metered-dose inhaler. 3. The patient will demonstrate the proper technique of self-administering a metered-dose inhaler. 4. The patient will comprehend the proper technique of self-administering a metered-dose inhaler.

3. The patient will demonstrate the proper technique of self-administering a metered-dose inhaler.

7. When the nurse considers the timing of a drug dose, which factor is appropriate to consider when deciding when to give a drug? 1. The patient's ability to swallow 2. The patient's height 3. The patient's last meal 4. The patient's allergies

3. The patient's last meal

18. The nurse is evaluating the effectiveness of guided imagery for pain management as used for a patient who has second- and third-degree burns and needs extensive dressing changes. Which finding best indicates the effectiveness of guided imagery? 1. The patient's facial expressions are stoic during the procedure. 2. The patient rates pain during the dressing change as a 6 on a scale of 0 to 10. 3. The patient's need for analgesic medication decreases during the dressing changes. 4. The patient asks for pain medication during the dressing changes only once throughout the procedure.

3. The patient's need for analgesic medication decreases during the dressing changes.

5. The nurse is reviewing pharmacology terms for a group of newly graduated nurses. Which sentence defines a drug's half-life? 1. The time it takes for the drug to cause half of its therapeutic response 2. The time it takes for one half of the original amount of a drug to reach the target cells 3. The time it takes for one half of the original amount of a drug to be removed from the body 4. The time it takes for one half of the original amount of a drug to be absorbed into the circulation

3. The time it takes for one half of the original amount of a drug to be removed from the body

A nurse is caring for a patient who is Muslim and has diabetes. Which of the following items does the nurse need to remove from the meal tray when it is delivered to the patient? 1. Small container of vanilla ice cream 2. A dozen red grapes 3. Bacon and eggs 4. Garden salad with ranch dressing

3. bacon and eggs - Islam prohibits the consumption of pork.

2. When given an intravenous medication, the patient says to the nurse, ―I usually take pills. Why does this medication have to be given in the arm?‖ What is the nurse's best answer? 1. ―The medication will cause fewer adverse effects when given intravenously.‖ 2. ―The intravenous medication will have delayed absorption into the body's tissues.‖ 3. ―The action of the medication will begin sooner when given intravenously.‖ 4. ―There is a lower chance of allergic reactions when drugs are given intravenously.‖

3. ―The action of the medication will begin sooner when given intravenously.‖

A 50-year-old woman is recovering from a bilateral mastectomy. She refuses to eat, discourages visitors, and pays little attention to her appearance. One morning the nurse enters the room to see the patient with her hair combed and makeup applied. Which of the following is the best response from the nurse? 1. "What's the special occasion?" 2. "You must be feeling better today." 3. "This is the first time I've seen you look this good." 4. "I see that you've combed your hair and put on makeup."

4. "I see that you've combed your hair and put on makeup." When the nurse uses a matter-of-fact approach and acknowledges a change in the patient's behavior or appearance, it allows the patient to establish its meaning. Telling the patient she has never looked this good conveys criticism; making assumptions about it being a special occasion or about an obvious improvement in mood superimposes the nurse's opinion and limits the assessment

14. A patient diagnosed with type 2 diabetes 26 years ago is beginning to experience peripheral neuropathy in the feet and lower leg. The nurse is providing education to the patient to prevent injury to the feet by wearing shoes or slippers when walking. Which statement made by the nurse best explains the rationale for this instruction? 1. "Wearing shoes blocks pain perception and helps you adapt to pain, which ends up protecting your feet." 2. "Shoes provide nonpharmacological pain relief to people with diabetes and peripheral neuropathy." 3. "The neurological gates open when wearing shoes, which protects your feet." 4. "If you step on something without shoes, you might not feel it; this could possibly cause injury to your foot."

4. "If you step on something without shoes, you might not feel it; this could possibly cause injury to your foot."

5. When the nurse teaches a skill such as self-injection of insulin to the patient, what is the best way to set up the teaching/learning session? 1. Provide written pamphlets for instruction. 2. Show a video, and allow the patient to practice as needed on his own. 3. Verbally explain the procedure, and provide written handouts for reinforcement. 4. After demonstrating the procedure, allow the patient to do several return demonstrations.

4. After demonstrating the procedure, allow the patient to do several return demonstrations.

7. The nurse recognizes that an elderly patient may experience a reduction in the stomach's ability to produce hydrochloric acid. This change may result in which effect? 1. Delayed gastric emptying 2. Increased gastric acidity 3. Decreased gastrointestinal motility 4. Altered absorption of some drugs

4. Altered absorption of some drugs

2. The nurse is monitoring a patient who is in the 26th week of pregnancy and has developed gestational diabetes and pneumonia. She is given medications that pose a possible fetal risk, but the potential benefits may warrant the use of the medications in her situation. The nurse recognizes that these medications are in which U.S. Food and Drug Administration pregnancy safety category? 1. Category A 2. Category B 3. Category C 4. Category D

4. Category D

24. A nurse wants to reduce data entry errors on the computer system. Which action should the nurse take? 1. Use the same password all the time. 2. Share password with only one other staff member. 3. Print out and review computer nursing notes at home. 4. Chart on the computer immediately after care is provided.

4. Chart on the computer immediately after care is provided.

3. The nurse is setting up a teaching session with an 85-year-old patient who will be going home on anticoagulant therapy. Which educational strategy would reflect consideration of the age-related changes that may exist with this patient? 1. Show a video about anticoagulation therapy. 2. Present all the information in one session just before discharge. 3. Give the patient pamphlets about the medications to read at home. 4. Develop large-print handouts that reflect the verbal information presented.

4. Develop large-print handouts that reflect the verbal information presented.

7. When reviewing pediatric medication administration, the nurse recognizes that which type of medication error is most common with children? 1. Oral medication administration errors 2. Wrong route errors 3. Incorrect dosage form errors 4. Dosing errors

4. Dosing errors

10. A nurse wants to find the daily weights of a hospitalized patient. Which resource will the nurse consult? 1. Database 2. Progress notes 3. Patient care summary 4. Graphic record and flow sheet

4. Graphic record and flow sheet

19. A nurse is providing medication education to a patient who just started been prescribed ibuprofen. Which information will the nurse include in the teaching session? 1. Ibuprofen helps to depress the central nervous system to decrease pain perception. 2. Ibuprofen reduces anxiety, which will help you cope with your pain. 3. Ibuprofen binds with opiate receptors to reduce your pain. 4. Ibuprofen inhibits the development of inflammation

4. Ibuprofen inhibits the development of inflammation

3. When given a scheduled morning medication, the patient states, I haven't seen that pill before. Are you sure it's correct? The nurse checks the medication administration record and verifies that it is listed. Which is the nurse's best response? 1. It's listed here on the medication sheet, so you should take it. 2. Go ahead and take it, and then I'll check with your doctor about it. 3. It wouldn't be listed here if it were not ordered for you! 4. Let me check on the order first before you take it.

4. Let me check on the order first before you take it.

3. When discussing dosage calculation for pediatric patients with a clinical pharmacist, the nurse notes that which type of dosage calculation is used most commonly in pediatric calculations? 1. West nomogram 2. Clark rule 3. Height-to-weight ratio 4. Milligram per kilogram of body weight formula

4. Milligram per kilogram of body weight formula

4. When monitoring the patient receiving an intravenous infusion to reduce blood pressure, the nurse notes that the patient's blood pressure is extremely low, and the patient is lethargic and difficult to awaken. This would be classified as which type of adverse drug reaction? 1. Adverse effect 2. Allergic reaction 3. Idiosyncratic reaction 4. Pharmacologic reaction

4. Pharmacologic reaction

8. A woman is in labor and refuses to receive any sort of anesthesia medication. Which alternative treatment is best for this patient? 1. Transcutaneous electrical nerve stimulation (TENS) 2. Herbal supplements with analgesic effects 3. Pudendal block (regional anesthesia) 4. Relaxation and guided imagery

4. Relaxation and guided imagery

19. A nurse is preparing to document a patient who has reported chest pain. Which information provided by the patient is critical for the nurse to include? 1. "My family doesn't believe I'm in pain." 2. Pupils equal and reactive to light. 3. Had poor results from the pain medication. 4. Reports sharp pain of 8 on a scale of 1 to 10.

4. Reports sharp pain of 8 on a scale of 1 to 10.

15. A home health nurse is preparing for an initial home visit. Which information should be included in the patient's home care medical record? 1. Nursing process form 2. Step-by-step skills manual 3. A list of possible procedures 4. Reports to third-party payers

4. Reports to third-party payers

9. An older-adult patient has extensive wound care needs after discharge from the hospital. Which facility should the nurse discuss with the patient? 1. Hospice 2. Respite care 3. Assisted living 4. Skilled nursing

4. Skilled nursing

4. After providing care, a nurse charts in the patient's record. Which entry will the nurse document? 1. Appears restless when sitting in the chair. 2. Drank adequate amounts of water. 3. Apparently is asleep with eyes closed. 4. Skin pale and cool.

4. Skin pale and cool.

10. A nurse working in a community hospital's emergency department provides care to a patient having chest pain. Which level of care is the nurse providing? 1. Continuing care 2. Restorative care 3. Preventive care 4. Tertiary care

4. Tertiary care

7. When reviewing the mechanism of action of a specific drug, the nurse reads that the drug works by selective enzyme interaction. Which of these processes describes selective enzyme interaction? 1. The drug alters cell membrane permeability. 2. The drug's effectiveness within the cell walls of the target tissue is enhanced. 3. The drug is attracted to a receptor on the cell wall, preventing an enzyme from binding to that receptor. 4. The drug binds to an enzyme molecule and inhibits or enhances the enzyme's action with the normal target cell.

4. The drug binds to an enzyme molecule and inhibits or enhances the enzyme's action with the normal target cell.

8. When administering a new medication to a patient, the nurse reads that it is highly protein bound. Assuming that the patient's albumin levels are normal, the nurse would expect which result, as compared to a medication, that is not highly protein bound? 1. Renal excretion will be faster. 2. The drug will be metabolized quickly. 3. The duration of action of the medication will be shorter. 4. The duration of action of the medication will be longer.

4. The duration of action of the medication will be longer.

6. The nurse is assessing an elderly Hispanic woman who is being treated for hypertension. During the assessment, what is important for the nurse to remember about cultural aspects? 1. The patient should be discouraged from using folk remedies and rituals. 2. The nurse will expect the patient to value protective bracelets and ―root doctors‖ as healers. 3. The nurse will remember that the balance among body, mind, and environment is important for this patient's health beliefs. 4. The nurse's assessment needs to include gathering information regarding religious practices and beliefs regarding medication, treatment, and healing.

4. The nurse's assessment needs to include gathering information regarding religious practices and beliefs regarding medication, treatment, and healing.

4. The nurse is assessing a newly admitted 83-year-old patient and determines that the patient is experiencing polypharmacy. Which statement most accurately illustrates polypharmacy? 1. The patient is experiencing multiple illnesses. 2. The patient uses one medication for an illness several times per day. 3. The patient uses over-the-counter drugs for an illness. 4. The patient uses multiple medications simultaneously.

4. The patient uses multiple medications simultaneously.

27. The nurse is administering pain medication for several patients. Which patient does the nurse administer medication to first? 1. The patient who needs to be premedicated before walking. 2. The patient who has a PCA running that needs the syringe replaced. 3. The patient who needs to take a scheduled dose of maintenance pain medication. 4. The patient who is experiencing 8/10 pain and has an immediate order for pain medication.

4. The patient who is experiencing 8/10 pain and has an immediate order for pain medication.

4. The nurse is assigned to a patient who is newly diagnosed with type 1 diabetes mellitus. Which statement best illustrates an outcome criterion for this patient? 1. The patient will follow instructions. 2. The patient will not experience complications. 3. The patient will adhere to the new insulin treatment regimen. 4. The patient will demonstrate correct blood glucose testing technique.

4. The patient will demonstrate correct blood glucose testing technique.

10. An elderly patient with a new diagnosis of hypertension will be receiving a new prescription for an antihypertensive drug. The nurse expects which type of dosing to occur with this drug therapy? 1. Drug therapy will be based on the patient's weight. 2. Drug therapy will be based on the patient's age. 3. The patient will receive the maximum dose that is expected to reduce the blood pressure. 4. The patient will receive the lowest possible dose at first, and then the dose will be increased as needed.

4. The patient will receive the lowest possible dose at first, and then the dose will be increased as needed.

2. A nurse is teaching the staff about integrated health care systems. Which model of care should the nurse include in the teaching about seam-less care delivery? 1. Affordable Care Act 2. Hospital Value-Based Purchasing 3. Bundled Payments for Care Improvements 4. The patient-centered medical home model

4. The patient-centered medical home model

6. When administering drugs, the nurse remembers that the duration of action of a drug is defined as which of these? 1. The time it takes for a drug to elicit a therapeutic response 2. The amount of time needed to remove a drug from circulation 3. The time it takes for a drug to achieve its maximum therapeutic response 4. The time period at which a drug's concentration is sufficient to cause a therapeutic response

4. The time period at which a drug's concentration is sufficient to cause a therapeutic response

11. The nurse is reviewing a list of scheduled drugs and notes that Schedule C-I drugs are not on the list. Which is a characteristic of Schedule C-I drugs? 1. No refills are permitted. 2. They may be obtained over-the-counter with a signature. 3. They are available only by written prescription. 4. They are used only with approved protocols.

4. They are used only with approved protocols.

A nurse uses the accepted rights of delegation when providing care. Which "rights" did the nurse use? (Select all that apply.) A. Task B. Person C. Direction D. Supervision E. Circumstances F. Cost-effectiveness

A, B, C, D, E

Which evaluative measures will the nurse use to determine a patient's responses to nursing care? (Select all that apply.) A. Observations of wound healing B. Daily blood pressure measurements C. Findings of respiratory rate and depth D. Completion of nursing interventions E. Patient's subjective report of feelings about a new diagnosis of cancer

A, B, C, E

A trauma survivor is requesting sleep medication because of "bad dreams." The nurse is concerned that the patient may be experiencing posttraumatic stress disorder (PTSD). Which question is a priority for the nurse to ask the patient? A. "Are you reliving your trauma?" B. "Are you having chest pain?" C. "Can you describe your phobias?" D. "Can you tell me when you wake up?"

A. "Are you reliving your trauma?"

A nurse assesses that a patient has not voided in 6 hours. Which question should the nurse ask to assist in establishing a nursing diagnosis of Urinary retention? A. "Do you feel like you need to go to the bathroom?" B. "Are you able to walk to the bathroom by yourself?" C. "When was the last time you took your medicine?" D. "Do you have a safety rail in your bathroom at home?"

A. "Do you feel like you need to go to the bathroom?"

A staff development nurse is providing an inservice for other nurses to educate them about the Nursing Interventions Classification (NIC) system. During the inservice, which statement made by one of the nurses in the room requires the staff development nurse to clarify the information provided? A. "This system can help medical students determine the cost of the care they provide to patients." B. "If the nursing department uses this system, communication among nurses who work throughout the hospital may be enhanced." C. "We could use this system to help organize orientation for new nursing employees because we can better explain the nursing interventions we use most frequently on our unit." D. "The NIC system provides one way to improve safe and effective documentation in the hospital's electronic health record."

A. "This system can help medical students determine the cost of the care they provide to patients."

A nurse is developing nursing diagnoses for a patient. Beginning with the first step, place in order the steps the nurse will use. 1. Observes the patient having dyspnea (shortness of breath) and a diagnosis of asthma 2. Writes a diagnostic label of impaired gas exchange. 3. Organizes data into meaningful clusters. 4. Interprets information from patient. 5. Writes an etiology. A. 1, 3, 4, 2, 5 B. 1, 3, 4, 5, 2 C. 1, 4, 3, 5, 2 D. 1, 4, 3, 2, 5

A. 1, 3, 4, 2, 5

The nurse is revising the care plan. In which order will the nurse perform the tasks, beginning with the first step? 1. Revise specific interventions. 2. Revise the assessment column. 3. Choose the evaluation method. 4. Delete irrelevant nursing diagnoses. A. 2, 4, 1, 3 B. 4, 2, 1, 3 C. 3, 4, 2, 1 D. 4, 2, 3, 1

A. 2, 4, 1, 3

A nurse is teaching the staff about the general adaptation syndrome. In which order will the nurse list the stages, beginning with the first stage? 1. Resistance 2. Exhaustion 3. Alarm A. 3, 1, 2 B. 3, 2, 1 C. 1, 3, 2 D. 1, 2, 3

A. 3, 1, 2

The standing orders for a patient include acetaminophen 650 mg every 4 hours prn for headache. After assessing the patient, the nurse identifies the need for headache relief and determines that the patient has not had acetaminophen in the past 4 hours. Which action will the nurse take next? A. Administer the acetaminophen. B. Notify the health care provider to obtain a verbal order. C. Direct the nursing assistive personnel to give the acetaminophen. D. Perform a pain assessment only after administering the acetaminophen.

A. Administer the acetaminophen.

A patient with an indwelling urinary catheter has been given a bed bath by a new nursing assistive personnel (NAP). The nurse evaluating the cleanliness of the patient notices crusting at the urinary meatus. Which action should the nurse take next? A. Asks the nursing assistive personnel to observe while the nurse performs catheter care. B. Leaves the room and ask the nursing assistive personnel to go back and perform proper catheter care. C. Tells the nursing assistive personnel that the incident will be reported to the nurse manager. D. Remove the catheter.

A. Asks the nursing assistive personnel to observe while the nurse performs catheter care.

The nurse is using critical thinking skills during the first phase of the nursing process. Which action indicates the nurse is in the first phase? A. Completes a comprehensive database. B. Identifies pertinent nursing diagnoses. C. Intervenes based on priorities of patient care. D. Determines whether outcomes have been achieved.

A. Completes a comprehensive database.

A nursing instructor needs to evaluate students' abilities to synthesize data and identify relationships between nursing diagnoses. Which learning assignment is best suited for this instructor's needs? A. Concept mapping B. Reflective journaling C. Lecture and discussion D. Reading assignment with a written summary

A. Concept mapping

While interviewing an older female patient of Asian descent, the nurse notices that the patient looks at the ground when answering questions. What should the nurse do? A. Consider cultural differences during this assessment. B. Ask the patient to make eye contact to determine her affect. C. Continue with the interview and document that the patient is depressed. D. Notify the health care provider to recommend a psychological evaluation.

A. Consider cultural differences during this assessment.

A home health nurse notices that a patient's preschool children are often playing on the sidewalk and in the street unsupervised and repeatedly takes them back to the home and talks with the patient, but the situation continues. Which immediate action by the nurse is mandated by law? A. Contact the appropriate community child protection facility. B. Tell the parents that the authorities will be contacted shortly. C. Take pictures of the children to support the overt child abuse. D. Discuss with both parents about the safety needs of their children.

A. Contact the appropriate community child protection facility.

1. Which are appropriate considerations when the nurse is assessing the learning needs of a patient? (Select all that apply.) A. Cultural background B. Family history C. Level of education D. Readiness to learn E. Health beliefs

A. Cultural background C. Level of education D. Readiness to learn E. Health beliefs

A nurse adds a nursing diagnosis to a patient's care plan. Which information did the nurse document? A. Decreased cardiac output related to altered myocardial contractility. B. Patient needs a low-fat diet related to inadequate heart perfusion. C. Offer a low-fat diet because of heart problems. D. Acute heart pain related to discomfort.

A. Decreased cardiac output related to altered myocardial contractility.

2. The nurse is teaching an older patient about the use of an incentive spirometer after surgery. Which of these age-related changes are appropriate for the nurse to consider when teaching older patients? (Select all that apply.) A. Decreased sense of touch B. Increased conduction of sound C. Decreased cognitive function D. Decreased short-term memory E. Increased ability to concentrate

A. Decreased sense of touch C. Decreased cognitive function D. Decreased short-term memory

Which action indicates a nurse is using critical thinking for implementation of nursing care to patients? A. Determines whether an intervention is correct and appropriate for the given situation. B. Reads over the steps and performs a procedure despite lack of clinical competency. C. Establishes goals for a particular patient without assessment. D. Evaluates the effectiveness of interventions.

A. Determines whether an intervention is correct and appropriate for the given situation.

A nurse is caring for a dying patient. When is the best time for the nurse to discuss end-of-life care? A. During assessment B. During planning C. During implementation D. During evaluation

A. During assessment

The nurse is reviewing a patient's plan of care, which includes the nursing diagnostic statement, Impaired physical mobility related to tibial fracture as evidenced by patient's inability to ambulate. Which part of the diagnostic statement does the nurse need to revise? A. Etiology B. Nursing diagnosis C. Collaborative problem D. Defining characteristic

A. Etiology

A patient continues to report postsurgical incision pain at a level of 9 out of 10 after pain medicine is given. The next dose of pain medicine is not due for another hour. What should the critically thinking nurse do first? A. Explore other options for pain relief. B. Discuss the surgical procedure and reason for the pain. C. Explain to the patient that nothing else has been ordered. D. Offer to notify the health care provider after morning rounds are completed.

A. Explore other options for pain relief.

A nurse identifies a fall risk when assessing a patient upon admission. The nurse and the patient agree that the goal is for the patient to remain free from falls. However, the patient fell just before shift change. Which action is the nurse's priority when evaluating the patient after the fall? A. Identifying factors interfering with goal achievement B. Counseling the nursing assistive personnel on duty when the patient fell C. Removing the fall risk sign from the patient's door because the patient has suffered a fall D. Requesting that the more experienced charge nurse complete the documentation about the fall

A. Identifying factors interfering with goal achievement

A hospital's wound nurse consultant made a recommendation for nurses on the unit about how to care for the patient's dressing changes. Which action should the nurses take next? A. Include dressing change instructions and frequency in the care plan. B. Assume that the wound nurse will perform all dressing changes. C. Request that the health care provider look at the wound. D. Encourage the patient to perform the dressing changes.

A. Include dressing change instructions and frequency in the care plan.

A nurse is providing postmortem care. Which action will the nurse take? A. Leave dentures in the mouth. B. Lower the head of the bed. C. Cover the body with a sterile sheet. D. Remove all tubes for an autopsy.

A. Leave dentures in the mouth.

A nurse is making a home visit and discovers that a patient's wound infection has gotten worse. The nurse cleans and redresses the wound. What should the nurse do next? A. Notify the health care provider of the findings before leaving the home. B. Ask the home health facility nurse manager to contact the health care provider. C. Document the findings and confirm with the patient the date of the next home visit. D. Tell the patient that the health care provider will be notified before the next home visit.

A. Notify the health care provider of the findings before leaving the home.

A nurse is assigned to care for the following patients who all need vital signs scheduled to be taken now. Which patient is most appropriate for the nurse to delegate vital sign measurement to the nursing assistive personnel (NAP)? A. Patient scheduled for a procedure in the nuclear. medicine department B. Patient transferring from the intensive care unit (ICU) C. Patient returning from a cardiac catheterization D. Patient returning from hip replacement surgery

A. Patient scheduled for a procedure in the nuclear. medicine department

A nurse is caring for a patient with a nursing diagnosis of Constipation related to slowed gastrointestinal motility secondary to pain medications. Which outcome is most appropriate for the nurse to include in the plan of care? A. Patient will have one soft, formed bowel movement by end of shift. B. Patient will walk unassisted to bathroom by the end of shift. C. Patient will be offered laxatives or stool softeners this shift. D. Patient will not take any pain medications this shift.

A. Patient will have one soft, formed bowel movement by end of shift.

A nurse is developing a care plan for a patient prescribed bed rest as a result of a pelvic fracture. Which goal statement is realistic for the nurse to assign to this patient? A. Patient will increase activity level this shift. B. Patient will turn side to back to side with assistance every 2 hours. C. Patient will use the walker correctly to ambulate to the bathroom as needed. D. Patient will use a sliding board correctly to transfer to the bedside commode as needed.

A. Patient will increase activity level this shift.

Which action by the nurse indicates a safe and efficient use of social networks? A. Promotes support for a local health charity. B. Posts a picture of a patient's infected foot. C. Vents about a patient problem at work. D. Friends a patient.

A. Promotes support for a local health charity.

A nurse is developing a care plan. Which intervention is most appropriate for the nursing diagnostic statement Risk for loneliness related to impaired verbal communication? A. Provide the patient with a writing board each shift. B. Obtain an interpreter for the patient as soon as possible. C. Assist the patient in performing swallowing exercises each shift. D. Ask the family to provide a sitter to remain with the patient at all times.

A. Provide the patient with a writing board each shift.

A nurse is planning care for a patient with a nursing diagnosis of Impaired skin integrity. The patient needs many nursing interventions, including a dressing change, several intravenous antibiotics, and a walk. Which factors does the nurse consider when prioritizing interventions? (Select all that apply.) A. Rank all the patient's nursing diagnoses in order of priority. B. Do not change priorities once they've been established. C. Set priorities based solely on physiological factors. D. Consider time as an influencing factor. E. Utilize critical thinking.

A. Rank all the patient's nursing diagnoses in order of priority. D. Consider time as an influencing factor. E. Utilize critical thinking.

A nurse has instituted a turn schedule for a patient to prevent skin breakdown. Upon evaluation, the nurse finds that the patient has a stage II pressure ulcer on the buttocks. Which action will the nurse take next? A. Reassess the patient and situation. B. Revise the turning schedule to increase the frequency. C. Delegate turning to the nursing assistive personnel. D. Apply medication to the area of skin that is broken down.

A. Reassess the patient and situation.

Which initial intervention is most appropriate for a patient who has a new onset of chest pain? A. Reassess the patient. B. Notify the health care provider. C. Administer a prn medication for pain. D. Call radiology for a portable chest x-ray.

A. Reassess the patient.

The nurse finds it difficult to care for a patient whose advance directive states that no extraordinary resuscitation measures should be taken. Which step may help the nurse to find resolution in this assignment? A. Review one's own personal values. B. Call for an ethical committee consult. C. Decline the assignment on religious grounds. D. Convince the family to challenge the directive.

A. Review one's own personal values.

A patient has sued a post-surgical unit nurse who provided care after abdominal surgery with nursing malpractice. Which resource would be used to determine whether the nurse has acted in a prudent manner? A. Scope and Standards of Nursing Care B. The typical level of care provided by other unit nurses C. The testimony of the patient's primary health care provider D. Comparison of documentation of the care provided by the nurse to similar patients

A. Scope and Standards of Nursing Care

A nurse manager sent one of the staff nurses on the unit to a conference about new, evidence-based wound care techniques. The nurse manager asks the staff nurse to prepare a poster to present at the next unit meeting, which will be mandatory for all nursing staff on the unit. Which type of opportunity is the nurse manager providing for the staff? A. Staff education B. Interprofessional collaboration C. Providing a professional shared governance council D. Establishing a nursing practice committee

A. Staff education

A goal for a patient diagnosed with diabetes is to demonstrate effective coping skills. Which patient behavior will indicate to the nurse achievement of this outcome? A. States, "It really helps talking about my health with family and friends." B. Observed consuming high-carbohydrate foods when stressed. C. Expresses a dislikes with the support group meetings. D. Spends most of the day reading in bed.

A. States, "It really helps talking about my health with family and friends."

The nurse is intervening for a patient with a risk for a urinary infection. Which direct care nursing intervention is most appropriate? A. Teaches proper handwashing technique. B. Properly cleans the patient's toilet. C. Transports urine specimen to the lab. D. Informs the oncoming nurse during hand-off.

A. Teaches proper handwashing technique

1. Which findings will alert the nurse that stress is present when making a clinical decision? (Select all that apply.) A. Tense muscles B. Reactive responses C. Trouble concentrating D. Feeling very tired E.Managed emotions

A. Tense muscles B. Reactive responses C. Trouble concentrating D. Feeling very tired

A nurse performs cardiopulmonary resuscitation (CPR) on a 92 year old with brittle bones and breaks a rib during the procedure, which then punctures a lung. The patient recovers completely without any residual problems and sues the nurse for pain and suffering and for malpractice. Which key point will the prosecution attempt to prove against the nurse? A. The CPR procedure was done incorrectly. B. The patient would have died if nothing was done. C. The patient was resuscitated according to the policy. D. The older patient with brittle bones might sustain fractures when chest compressions are done.

A. The CPR procedure was done incorrectly.

Which scenario best illustrates the nurse using data validation when making a nursing clinical decision for a patient? A. The nurse determines to remove a wound dressing when the patient reveals the time of the last dressing change and notices old and new drainage. B. The nurse administers pain medicine due at 1700 at 1600 because the patient reports increased pain and the family wants something done. C. The nurse immediately asks the health care provider for an order of potassium when a patient reports leg cramps. D. The nurse elevates a leg cast when the patient reports decreased mobility.

A. The nurse determines to remove a wound dressing when the patient reveals the time of the last dressing change and notices old and new drainage.

2. The nurse is performing an admission assessment. Which findings reflect components of a cultural assessment? (Select all that apply.) A. The patient uses aspirin as needed for pain. B. The patient has a history of hypertension. C. The patient uses herbal tea to relax in the evenings. D. The patient does not speak English. E. The patient is allergic to shellfish. F. The patient does not eat pork products because of religious beliefs.

A. The patient uses aspirin as needed for pain. C. The patient uses herbal tea to relax in the evenings. D. The patient does not speak English. F. The patient does not eat pork products because of religious beliefs.

The following statements are on a patient's nursing care plan. When creating a nursing care plan, which statement should the nurse use as an outcome for a goal of care? A. The patient will verbalize a decreased pain level less than 3 on a 0 to 10 scale by the end of this shift. B. The patient will demonstrate increased tolerance to activity over the next month. C. The patient will understand needed dietary changes by discharge. D. The patient will demonstrate increased mobility in 2 days.

A. The patient will verbalize a decreased pain level less than 3 on a 0 to 10 scale by the end of this shift.

A nurse is helping parents who have a child with attention-deficit/hyperactivity disorder. Which strategy will the nurse share with the parents to reduce stress regarding homework assignments? A. Time-management skills B. Speech articulation skills C. Routine preventative health visits D. Assertiveness training for the family

A. Time-management skills

A patient has had two family members die during the past 2 days. Which coping strategy is most appropriate for the nurse to suggest to the patient? A. Writing in a journal B. Drinking a little alcohol to go to sleep C. Exercising vigorously rather than sleeping D. Avoiding talking about the deaths with friends

A. Writing in a journal

When caring for patients, the nurse must understand the difference between religion and spirituality. Religious care helps individuals: A. maintain their belief systems and worship practices. B. develop a relationship with a higher being. C. establish a cultural connectedness with the purpose of life. D. achieve the balance needed to maintain health and well-being.

A. maintain their belief systems and worship practices.

Four patients in labor all request epidural analgesia to manage their pain at the same time. Which ethical principle is most compromised when only one nurse anesthetist is on call? a. Justice b. Fidelity c. Beneficence d. Nonmaleficence

A: Justice

1. Which statements are true regarding pediatric patients and pharmacokinetics? (Select all that apply.) 1. The levels of microsomal enzymes are decreased. 2. Perfusion to the kidneys may be decreased and may result in reduced renal function. 3. First-pass elimination is increased because of higher portal circulation. 4. First-pass elimination is reduced because of the immaturity of the liver. 5. Total body water content is much less than in adults. 6. Gastric emptying is slowed because of slow or irregular peristalsis. 7. Gastric emptying is more rapid because of increased peristaltic activity.

ANS: 1, 2, 4, 5

A nurse is assessing a patient with prolonged stress. Which conditions will the nurse monitor for in this patient? (Select all that apply.) A. Cancer B. Diabetes C. Infections D. Allostasis E. Low blood pressure

ANS: A, B, C

A nurse is providing nursing care to a group of patients. Which actions are direct care interventions? (Select all that apply.) A. Ambulating a patient B. Inserting a feeding tube C. Performing resuscitation D. Documenting wound care E. Teaching about medications

ANS: A, B, C, E

Which interventions are appropriate for a patient with diabetes and poor wound healing? (Select all that apply.) A. Perform dressing changes twice a day as ordered. B. Teach the patient about signs and symptoms of infection. C. Instruct the family about how to perform dressing changes. D. Gently refocus patient from discussing body image changes. E. Administer medications to control the patient's blood sugar as ordered.

ANS: A, B, C, E

Etic worldview

An outsider's perspective

3. During a nursing assessment a patient displayed several behaviors. Which behavior suggests the patient may have a health literacy problem? 1. Patient has difficulty completing a registration form at a medical office 2. Patient asks for written information about a health topic 3. Patient speaks Spanish as primary language 4. Patient states unfamiliarity with a newly ordered medicine

Answer: 1. Behaviors that might reflect a health literacy deficit include having difficulty completing registration forms or health histories, failing to make follow-up appointments, and asking few questions during a nursing history or physical examination

9. Which statement made by a new graduate nurse about the teachback technique requires intervention and further instruction by the nurse's preceptor? 1. "After teaching a patient how to use an inhaler, I need to use the teach-back technique to test my patient's technique." 2. "The teach-back technique is an ongoing process of asking patients for feedback." 3. "Using teach-back will help me identify explanations and communication strategies that my patients will most commonly understand." 4. "Using pictures, drawings, and models can enhance the effectiveness of the teach-back technique."

Answer: 1. Teach-back is not a testing of patient knowledge or ability to use devices but is a confirmation of how well a nurse explains concepts to patients.

The nurse can increase a patient's self-awareness and self-concept through which of the following actions? (Select all that apply.) 1. Helping the patient define personal problems clearly 2. Allowing the patient to openly explore thoughts and feelings 3. Reframing the patient's thoughts and feelings in a more positive way 4. Having family members assume more responsibility during times of stress 5. Recommending self-help reading materials

Answer: 1, 2, 3. Helping a patient define problems clearly, allowing him or her to openly explore thoughts and feelings, and reframing his or her thoughts and feelings in a more positive way are designed to promote self-awareness and a positive self-concept. Having the family assume more responsibility does not help a patient achieve self-awareness; instead it is important to encourage the patient to assume more self-responsibility. The nurse should refrain from offering self-help reading materials unless directly asked; the nurse should then provide numerous options.

A 20-year-old patient diagnosed with an eating disorder has a nursing diagnosis of Situational Low Self-Esteem. Which of the following nursing interventions are appropriate to address self-esteem? (Select all that apply.) 1. Offer independent decision-making opportunities. 2. Review previously successful coping strategies. 3. Provide a quiet environment with minimal stimuli. 4. Support a dependent role throughout treatment. 5. Increase calorie intake to promote weight stabilization

Answer: 1, 2. Offering opportunities for decision making promotes a sense of control, which is essential for promoting independence and enhancing self-esteem. Reviewing successful coping strategies is also a priority intervention to signal previous mastery and promote effective coping in an individual with self-esteem issues. The amount of stimuli is unrelated to self-esteem. Promoting independence is an important part of treatment. Although weight stabilization may be needed, it is likely to have a negative effect on self-esteem early in treatment.

When assessing a patient's adjustment to the role changes brought about by a medical condition such as a stroke, the nurse asks about which of the following? (Select all that apply.) 1. What are your thoughts about returning to work? 2. What questions do you have about your medications? 3. How has your health affected your relationship with your partner? 4. What level of physical activity are you able to perform? 5. What concerns do you have about another stroke?

Answer: 1, 3. The nurse must assess role performance as related to professional identity (work) and personal relationships (partner). The other questions are important, but not related to self-concept.

The nurse is caring for a 50-year-old woman visiting the outpatient medicine clinic. The patient has had type 1 diabetes since age 13. She has numerous complications from her disease, including reduced vision, heart disease, and severe numbness and tingling of the extremities. Knowing that spirituality helps patients cope with chronic illness, which of the following principles should the nurse apply in practice? (Select all that apply.) 1. Pay attention to the patient's spiritual identity throughout the course of her illness. 2. Select interventions that you know scientifically support spiritual well-being. 3. Listen to the patient's story each visit to the clinic, and offer a compassionate presence. 4. When the patient questions the reason for her long-time suffering, try to provide answers. 5. Consult with a spiritual care adviser, and have the adviser recommend useful intervention

Answer: 1, 3. A person's spiritual well-being can change over time; therefore, it is important to pay attention to it over the course of his or her illness. Listening is a powerful way to support a patient's spirituality. Evidence-based interventions are preferred, but they must be agreed on by the patient and tailored to his or her perspectives and not just those of the nurse. Patients are not looking for answers. What is spoken as a spiritual question is most often an expression of spiritual pain. Using spiritual care advisers is a valuable resource but should be selected by the patient, not independently by the nurse, and any interventions should be mutually agreed on among nurse, patient, and adviser.

A nurse is working with an older adult who recently moved to an assisted-living center because of declining physical capabilities associated with the normal aging process. Which nursing interventions are directed at promoting self-esteem in this patient? 1. Commending the patient's efforts at completing self-care tasks 2. Assuming that the patient's physical complaints are attention-seeking measures 3. Minimizing time discussing memories and past achievements spent with the patient 4. Limiting decision-making opportunities for the patient to reduce stress

Answer: 1. Reinforce efforts to complete tasks, allowing additional time to complete tasks if needed and support efforts directed at independence. This fosters self-esteem and confidence. It is important for the nurse to refrain from assumptions, as in assuming that physical complaints are attention seeking or that limiting decisions will reduce stress. Time should be allocated to review of past accomplishments and memories.

8. Which explanation provided by the nurse is the most accurate meaning for "providing culturally congruent care"? 1. It fits the patient's valued life patterns and set of meanings. 2. It is the same set of values as those of the health care team member providing daily care. 3. It holds one's own way of life as superior to those of others. 4. It redirects the patient to a more socially expected set of values.

Answer: 1. The ongoing process of cultural competence refers to the need of the health care professional to be culturally sensitive in providing culturally appropriate care specific to the needs of the patient, family, and community.

The nurse is caring for a patient who is very depressed and decides to complete a spiritual assessment using the FICA tool. Using the FICA assessment tool, match the criteria on the left with the appropriate assessment question on the right. 1. F—Faith ___ 2. I—Importance of spirituality ___ 3. C—Community ___ 4. A—Interventions to address spiritual needs ___ a. Tell me if you have a higher power or authority that helps you act on your beliefs b. Describe which activities give you comfort spiritually c. To whom do you go for support in times of difficulty? d. Your illness has kept you from attending church. Is that a problem for you?

Answer: 1a, 2d, 3c, 4b.

A nurse is caring for a 40-year-old male diagnosed with Crohn's disease several years ago, resulting in numerous hospitalizations each year for the past 3 years. Which of the following behaviors interfere with the developmental tasks of middle adulthood? (Select all that apply.) 1. Sends birthday cards to friends and family 2. Refuses visitors while hospitalized 3. Self-absorbed in physical and psychological issues 4. Performs self-care activities 5. Communicates feelings of inadequacy

Answer: 2, 3, 5. Developmental tasks of adulthood can be impacted by chronic illness. Self-absorption and the refusal to stay connected with others are of concern to the nurse, as are verbalizations of inadequacy. Staying in touch with friends and performing self-care behaviors demonstrate developmental mastery of adulthood.

A nurse is preparing to teach an older adult who has chronic arthritis how to practice meditation. Which of the following strategies are appropriate? (Select all that apply.) 1. Encourage family members to participate in the exercise. 2. Have patient identify a quiet room in the home that has minimal interruptions. 3. Suggest the use of a quiet fan running in the room. 4. Explain that it is best to meditate about 5 minutes 4 times a day. 5. Show the patient how to sit comfortably with the limitation of his arthritis and focus on a prayer.

Answer: 2, 3, 5. A quiet room with no distractions is conducive to meditation. The low buzz of a fan also blocks distractive noises. A patient should relax comfortably during meditation. Meditation is usually recommended 10 to 20 minutes twice a day. The activity should be conducted alone without distraction.

Which of the following assessment findings suggest an altered self-concept? (Select all that apply.) 1. Uneven gait 2. Slumped posture and poor personal hygiene 3. Avoidance of eye contact when answering a question 4. Requests for visits from the chaplain 5. Frequent use of the call light

Answer: 2, 3. Common assessment findings for an individual with altered self-concept can mirror depressive symptoms, such as slumped posture, poor hygiene, and avoiding intermittent eye contact.An individual with an unsteady or uneven gait may have successfully adjusted to an underlying condition; this does not automatically signal an altered self-concept. Requests for spiritual support and nursing care should be honored and are not related to an altered self-concept.

. A 44-year-old male patient has just been told that his wife and child were killed in an auto accident while coming to visit him in the hospital. Which of the following statements are assessment findings that support a nursing diagnosis of Spiritual Distress related to loss of family members? (Select all that apply.) 1. "I need to call my sister for support." 2. "I have nothing to live for now." 3. "Why would my God do this to me?" 4. "I need to pray for a miracle." 5. "I want to be more involved in my church."

Answer: 2, 3. Patients most likely to have a diagnosis of Spiritual Distress are facing loss or terminal or serious illness and have poor personal relationships. Indicating that there is nothing to live for now and wondering why God would do this to him reflect dispiritedness (e.g., expressing lack of hope, meaning, or purpose in life; anger toward God). The other responses show a potential for enhancement of spiritual well-being.

A nursing student is developing a plan of care for a 74-year-old-female patient who has spiritual distress over losing a spouse. As the nurse develops appropriate interventions, which characteristics of older adults should be considered? (Select all that apply.) 1. Older adults do not routinely use complementary medicine to cope with illness. 2. Older adults dislike discussing the afterlife and what might have happened to people who have passed on. 3. Older adults achieve spiritual resilience through frequent expressions of gratitude. 4. Have the patient determine whether her husband left a legacy behind. 5. Offer the patient her choice of rituals or participation in exercise.

Answer: 3, 4, 5. Older-adult patients achieve spiritual resiliency in expressing gratitude and finding ways to maintain purpose in life. Leaving legacies maintains a connection between the person left behind and the lost loved one. Older adults frequently use complementary medicine, rituals, and exercise to cope with illness and pain. Belief in the afterlife grows with aging.

6. A nurse working in a large occupational health clinic knows that many of the workers at her company are marginalized and at risk for poor health outcomes. Which of the following individuals are most likely to be marginalized? 1. Wives of the employees 2. The head supervisors of the company 3. Workers who have a high school education 4. Workers employed for less than a year at the company

Answer: 3. Marginalized groups are more likely to have poor health outcomes and die earlier because of a complex interaction among their individual behaviors, environment of the communities in which they live, the policies and practices of health care and governmental systems, and the clinical care they receive. Examples of marginalized groups include people who are gay, lesbian, bisexual, or transgender; people of color; people who are physically and/ or mentally challenged; and people who are not college educated.

The home health nurse is visiting a 90-year-old man who lives with his 89-year-old wife. He is legally blind and is 3 weeks' post right hip replacement. He ambulates with difficulty with a walker. He comments that he is saddened now that his wife has to do more for him and he is doing less for her. Which of the following is the priority nursing diagnosis? 1. Impaired Self Toileting 2. Lack of Knowledge Regarding Resources for the Visually Impaired 3. Disturbed Body Image 4. Risk for Situational Low Self-Esteem

Answer: 4 Blindness coupled with difficulty ambulating places him at risk for situational low self-esteem. He and his wife most likely have adapted to the blindness, but his difficulty with ambulation affects many aspects of his life, including self-esteem. However, this low self-esteem is situational; as his mobility improves, his low self-esteem will also resolve. Nothing in the question itself suggests that the other diagnoses are true

7. A mother is concerned about her child's flulike symptoms. You learn from the health assessment that the mother practices use of "hot" and "cold" foods to treat ailments. Which of the following foods do you expect the mother to use to treat her child? 1. Chicken 2. Yogurt 3. Fresh fruits 4. Eggs

Answer: 4. Certain cultures believe in the importance of balance and harmony in health. Natural or holistic balance is believed to be achieved by using "hot" and "cold" foods as remedies to treat illness.

Which statement made by a patient who is recovering after recently experiencing third-degree burns shows connectedness? 1. "My pain medicine helps me feel better." 2. "I know I will get better if I just keep trying." 3. "I see God's grace and become relaxed when I watch the sun set at night." 4. "I feel so much closer to God after I read my Bible and pray."

Answer: 4. Connectedness is a dimension of spirituality that is related to the human need of belonging. Individuals can be connected to themselves, others, God or another Supreme Being, or nature. Individuals often stay connected to God through prayer.

5. Health care organizations must provide which of the following based on federal civil rights laws? (Select all that apply.) 1. Provide language assistance services at all points of contact free of charge. 2. Provide auxiliary aids and services, such as interpreters, note takers, and computer-aided transcription services. 3. Use patients' family members to interpret difficult topics. 4. Ensure that interpreters are competent in medical terminology. 5. Provide language assistance to all patients who speak limited English or are deaf.

Answers: 1, 2, 4, 5. The CLAS standards include standards for communication and language assistance, including providing language assistance free of charge, auxiliary aids and services, interpreters competent in medical terminology, and language assistance for patients with limited English or who are deaf

2. A 35-year-old woman has Medicaid coverage for herself and two young children. She missed an appointment at the local health clinic to get an annual mammogram because she has no transportation. She gets the annual screening because her mother had breast cancer. Which of the following are social determinants of this woman's health? (Select all that apply.) 1. Medicaid insurance 2. Annual screening 3. Mother's history of breast cancer 4. Lack of transportation5. Woman's age

Answers: 1, 4, 5. Social determinants of health are the conditions under which persons are born, grow, live, work, and age. The social determinants of health are mostly responsible for health disparities. Examples include age, race and ethnicity, socioeconomic status (as reflected by the woman's insurance), access to nutritious food, transportation resources, religion, sexual orientation, level of education, literacy level, disability (physical and cognitive), and geographic location (e.g., access to health care).

10. Match the cultural concepts on the left with the correct definitions on the right. 1. Etic world view __________ 2. World view __________ 3. Cultural desire __________ 4. Intersectionality __________ 5. Emic world view __________ a. Factor that shapes how people perceive others and how they relate to reality b. Insider's perspective in an intercultural encounter c. A policy model that describes factors and power structures that shape and influence life d. An outsider's perspective in an intercultural encounter e. The motivation of a health care professional to "want to" engage in cultural competency

Answers: 1d, 2a, 3e, 4c, 5b.

4. A nurse desires to communicate with a young woman who is Serbian and who has limited experience with being in a hospital. The nurse has 10 years of experience caring for Serbian women. The patient was admitted for a serious pregnancy complication. Apply the LEARN model and match the nurse's behaviors with each step of the model. 1. L ___________ 2. E ___________ 3. A ___________ 4. R __________ 5. N __________ a. The nurse notes that she has learned that fathers can visit mothers at any time in both Serbia and the United States. b. The nurse shares her perception of the woman's experiences as a patient. c. The nurse asks the patient how she can maintain bed rest when she returns home. d. The nurse attends to the patient and listens to her story about hospitals in Serbia. e. The nurse involves the patient in a discussion of the treatment options for her condition.

Answers: 1d, 2b, 3a, 4e, 5c. Listen with empathy and understanding to patient perception of the problem; Explain your perceptions of the problem (physiological, psychological, spiritual, and/or cultural; Acknowledge and discuss cultural differences and similarities between you and your patient; Recommend treatment (involving the patient); and Negotiate agreement (incorporate selected aspects of the patient's culture into patient-centered care).

A novice nurse is confused about using evaluative measures when caring for patients and asks the charge nurse for an explanation. Which response by the charge nurse is most accurate? A. "Evaluative measures are multiple-page documents used to evaluate nurse performance." B. "Evaluative measures include assessment data used to determine whether patients have met their expected outcomes and goals." C. "Evaluative measures are used by quality assurance nurses to determine the progress a nurse is making from novice to expert nurse." D. "Evaluative measures are objective views for completion of nursing interventions."

B. "Evaluative measures include assessment data used to determine whether patients have met their expected outcomes and goals."

An Orthodox Jewish rabbi has been pronounced dead. The nursing assistive personnel respectfully ask family members to leave the room and go home as postmortem care is provided. Which statement from the supervising nurse is best? A. "I should have called a male colleague to handle the body." B. "Family members stay with the body as part of the mourning ritual." C. "I wish they would go home because we have work to do here." D. "Family will quietly leave after praying and touching the rabbi's head."

B. "Family members stay with the body as part of the mourning ritual."

A senior college student visits the college health clinic about a freshman student living on the same dormitory floor. The senior student reports that the freshman is crying and is not adjusting to college life. The clinic nurse recognizes this as a combination of situational and maturational stress factors. Which is the best response by the nurse? A. "Let's call 9-1-1 because this freshman student is suicidal." B. "Give the freshman student this list of university and community resources." C. "I recommend that you help the freshman student start packing bags to go home." D. "You must make an appointment for the freshman student to obtain medications."

B. "Give the freshman student this list of university and community resources."

Which question would be most appropriate for a nurse to ask a patient to assist in establishing a nursing diagnosis of Diarrhea? A. "What types of foods do you think caused your upset stomach?" B. "How many bowel movements a day have you had?" C. "Are you able to get to the bathroom in time?" D. "What medications are you currently taking?"

B. "How many bowel movements a day have you had?"

While recovering from a severe illness, a hospitalized patient wants to change a living will, which was signed 9 months ago. Which response by the nurse is most appropriate? A. "Check with your admitting health care provider whether a copy is on your chart." B. "Let me check with someone here in the hospital who can assist you." C. "You are not allowed to ever change a living will after signing it." D. "Your living will can be changed only once each calendar year."

B. "Let me check with someone here in the hospital who can assist you."

After a natural disaster occurred, an emergency worker referred a family for crisis intervention services. One family member refused to attend the services, stating, "No way, I'm not crazy." What is the nurse's best response? A. "Many times, disasters can create mental health problems, so you really should participate with your family." B. "Seeking this kind of help does not mean that you have a mental illness; it is a short-term problem-solving technique." C. "Don't worry now. The psychiatrists are well trained to help." D. "This will help your family communicate better."

B. "Seeking this kind of help does not mean that you have a mental illness; it is a short-term problem-solving technique."

The nurse is attempting to prompt the patient to elaborate on the reports of daytime fatigue. Which question should the nurse ask? A. "Is there anything that you are stressed about right now that I should know?" B. "What reasons do you think are contributing to your fatigue?" C. "What are your normal work hours?" D. "Are you sleeping 8 hours a night?"

B. "What reasons do you think are contributing to your fatigue?"

A nursing student has been written up several times for being late with providing patient care and for omitting aspects of patient care and not knowing basic procedures that were taught in the skills course one term earlier. The nursing student says, "I don't understand what the big deal is. As my instructor, you are there to protect me and make sure I don't make mistakes." What is the best response from the nursing instructor? A. "You are practicing under the license of the hospital's insurance." B. "You are expected to perform at the level of a professional nurse." C. "You are expected to perform at the level of a prudent nursing student." D. "You are practicing under the license of the nurse assigned to the patient."

B. "You are expected to perform at the level of a professional nurse."

A nurse is modifying a patient's care plan after evaluation of patient care. In which order, starting with the first step, will the nurse perform the tasks? 1. Revise nursing diagnosis. 2. Reassess blood pressure reading. 3. Retake blood pressure after medication. 4. Administer new blood pressure medication.5. Change goal to blood pressure less than 140/90. A. 1, 5, 2, 4, 3 B. 2, 1, 5, 4, 3 C. 4, 3, 1, 5, 2 D. 5, 4, 5, 1, 2

B. 2, 1, 5, 4, 3

Which diagnosis will the nurse document in a patient's care plan that is NANDA-I approved? A. Sore throat B. Acute pain C. Sleep apnea D. Heart failure

B. Acute pain

While caring for a hospitalized older-adult female post hip surgery, the nurse is faced with the task of inserting an indwelling urinary catheter, which involves rotating the hip into a contraindicated position. Which action should the nurse take? A.Postpone catheter insertion until the next shift. B. Adapt the positioning technique to the situation. C. Notify the health care provider for a urologist consult. D. Follow textbook procedure with contraindicated position.

B. Adapt the positioning technique to the situation.

A nurse has already set the agenda during a patient-centered interview. What will the nurse do next? A. Begin with introductions. B. Ask about the chief concerns or problems. C. Explain that the interview will be over in a few minutes. D. Tell the patient "I will be back to administer medications in 1 hour."

B. Ask about the chief concerns or problems.

While completing an admission database, the nurse is interviewing a patient who states, "I am allergic to latex." Which action will the nurse take first? A. Immediately place the patient in isolation. B. Ask the patient to describe the type of reaction. C. Proceed to the termination phase of the interview. D. Document the latex allergy on the medication administration record.

B. Ask the patient to describe the type of reaction.

A patient visiting with family members in the waiting area tells the nurse "I don't feel good, especially in the stomach." What should the nurse do? A. Request that the family leave, so the patient can rest. B. Ask the patient to return to the room, so the nurse can inspect the abdomen. C. Ask the patient when the last bowel movement was and to lie down on the sofa. D. Tell the patient that the dinner tray will be ready in 15 minutes and that may help the stomach feel better.

B. Ask the patient to return to the room, so the nurse can inspect the abdomen.

A nurse is pulled from the surgical unit to work on the oncology unit. Which action by the nurse displays humility and responsibility? A. Refusing the assignment B. Asking for an orientation to the unit C. Admitting lack of knowledge and going home D. Assuming that patient care will be the same as on the other units

B. Asking for an orientation to the unit

How can a nurse assigned to a medical unit at a local hospital best address issues related to the delivery of quality nursing care? A. Serve as a volunteer patient advocate at the local free health clinic. B. Become active in professional nursing organizations at the state level. C. Ask to be a member of the hospital's policy and procedure committee D. Agree to act as a preceptor for nursing students during their clinical experience

B. Become active in professional nursing organizations at the state level.

Which action will the nurse take after the plan of care for a patient is developed? A. Placing the original copy in the chart, so it cannot be tampered with or revised B. Communicating the plan to all health care professionals involved in the patient's care C. Filing the plan of care in the administration office for legal examination D. Sending the plan of care to quality assurance for review

B. Communicating the plan to all health care professionals involved in the patient's care

A patient is having difficulty reaching the water fountain while holding on to crutches. The nurse suggests that the patient place the crutches against the wall while stabilizing him or herself with two hands on the water fountain. Which critical thinking attitude did the nurse use in this situation? A. Humility B. Creativity C. Risk taking D. Confidence

B. Creativity

The patient database reveals that a patient has decreased oral intake, decreased oxygen saturation when ambulating, reports of shortness of breath when getting out of bed, and a productive cough. Which elements will the nurse identify as defining characteristics for the diagnostic label of Activity intolerance? A. Decreased oral intake and decreased oxygen saturation when ambulating B. Decreased oxygen saturation when ambulating and reports of shortness of breath when getting out of bed C. Reports of shortness of breath when getting out of bed and a productive cough D. Productive cough and decreased oral intake

B. Decreased oxygen saturation when ambulating and reports of shortness of breath when getting out of bed

A novice nurse is working in a unit that uses interdisciplinary collaboration. Which action will the nurse take to help assure effectiveness as a team member? A. Act as a leader of the health care team. B. Develop good communication skills. C. Work solely with experienced nurses. D. Avoid conflict.

B. Develop good communication skills.

A nursing assistive personnel (NAP) reports seeing a reddened area on the patient's hip while bathing the patient. Which action should the nurse take? A. Request a wound nurse consult. B. Go to the patient's room to assess the patient's skin. C. Document the finding per the NAP's report. D. Ask the NAP to apply a dressing over the reddened area.

B. Go to the patient's room to assess the patient's skin.

A nurse is providing education to a patient about self-administering subcutaneous injections. The patient demonstrates the self-injection. Which type of indicator did the nurse evaluate? A. Health status B. Health behavior C. Psychological self-control D. Health service utilization

B. Health behavior

A new nurse writes the following nursing diagnoses on a patient's care plan. Which nursing diagnosis will cause the nurse manager to intervene? A. Wandering B. Hemorrhage C. Urinary retention D. Impaired swallowing

B. Hemorrhage

1. The nurse is reviewing the concept of drug polymorphism. Which factors contribute to drug polymorphism? (Select all that apply.) A. The number of drugs ordered by the physician B. Inherited factors C. The patient's diet and nutritional status D. Different dosage forms of the same drug E. The patient's cultural practices F. The patient's drug history G. The various available forms of a drug

B. Inherited factors C. The patient's diet and nutritional status E. The patient's cultural practices

A nurse is teaching a patient and family about quality of life. Which information should the nurse include in the teaching session about quality of life? A. It is deeply social. B. It is hard to define and deeply personal C. It is an observed measurement for most people. D. It is consistent and stable over the course of one's lifetime.

B. It is hard to define and deeply personal

A nursing assistive personnel (NAP) is caring for a dying patient. Which action by the NAP will cause the nurse to intervene? A. Elevating head of bed B. Making the patient eat C. Giving mouth care every 2 to 4 hours D. Keeping skin clean, dry, and moisturized

B. Making the patient eat

A nurse is reviewing care plans. Which finding, if identified in a plan of care, should the registered nurse revise? A. Patient's outcomes for learning B. Nurse's assumptions about hospital discharge C. Identification of several actual health problems D. Documentation of patient's ability to meet the goal

B. Nurse's assumptions about hospital discharge

A nurse is conducting a nursing health history. Which component will the nurse address? A. Nurse's concerns B. Patient expectations C. Current treatment orders D. Nurse's goals for the patient

B. Patient expectations

A woman who was sexually assaulted a month ago presents to the emergency department with reports of recurrent nightmares, fear of going to sleep, repeated vivid memories of the sexual assault, and inability to feel much emotion. Which medical problem will the nurse expect to see documented in the chart? A. General adaptation syndrome (GAS) B. Posttraumatic stress disorder (PTSD) C. Acute stress disorder D. Alarm reaction

B. Posttraumatic stress disorder (PTSD)

A nurse is experiencing an ethical dilemma with a patient. Recognizing what information as a factor indicates the nurse has a correct understanding of the primary cause of ethical dilemmas? A. Unequal power B. Presence of conflicting values C. Judgmental perceptions of patients D. Poor communication with the patient

B. Presence of conflicting values

The nurse is teaching a novice nurse about protocols. Which information from the novice nurse indicates a correct understanding of the teaching? A. Protocols are guidelines to follow that replace the nursing care plan. B. Protocols assist the clinician in making decisions and choosing interventions for specific health care problems or conditions. C. Protocols are policies designating each nurse's duty according to standards of care and a code of ethics. D. Protocols are prescriptive order forms that help individualize the plan of care.

B. Protocols assist the clinician in making decisions and choosing interventions for specific health care problems or conditions.

A previously toilet trained toddler has started wetting again. A nurse is gathering a health history from the grandparent. Which health history finding will the nurse most likely consider as the cause of the wetting? A. Dietary changes B. Recent parental death C. Playmate moved away D. Sibling was sick 2 days

B. Recent parental death

A recent immigrant who does not speak English is alert but requires hospitalization. What is the initial action that the nurse must take to enable informed consent to be obtained? A. Ask a family member to translate what the nurse is saying. B. Request an official interpreter to explain the terms of consent. C. Notify the nursing manager that the patient doesn't speak English. D. Use hand gestures and medical equipment while explaining in English

B. Request an official interpreter to explain the terms of consent.

The nurse is caring for a patient whose plan of care states that a change of dressing is to occur twice a day, at 0600 and 1800. At 1400, the nurse notices that the dressing is saturated and leaking. What is the nurse's next action? A. Wait and change the dressing at 1800 as ordered. B. Revise the plan of care and change the dressing now. C. Reassess the dressing and the wound in 2 hours. D. Discontinue the plan of care for wound care.

B. Revise the plan of care and change the dressing now.

A veteran is hospitalized after surgical amputation of both lower extremities owing to injuries sustained during military service. Which type of loss will the nurse focus the plan of care on for this patient? A. Perceived loss B. Situational loss C. Maturational loss D. Uncomplicated loss

B. Situational loss

When professionals work together to solve ethical dilemmas, nurses must examine their own values. What is the best rationale for this step? A. So fact is separated from opinion. B. So different perspectives are respected. C. So judgmental attitudes can be provoked. D. So the group identifies the one correct solution.

B. So different perspectives are respected.

Despite working in a highly stressful nursing unit and accepting additional shifts, a new nurse has a strategy to prevent burnout. Which strategy will be best for the nurse to use? A. Delegate complex nursing tasks to nursing assistive personnel. B. Strengthen friendships outside the workplace. C. Write for 10 minutes in a journal every day. D. Use progressive muscle relaxation.

B. Strengthen friendships outside the workplace.

Which action indicates a registered nurse is being responsible for making clinical decisions? A. Applies clear textbook solutions to patients' problems. B. Takes immediate action when a patient's condition worsens. C. Uses only traditional methods of providing care to patients. D. Formulates standardized care plans solely for groups of patients.

B. Takes immediate action when a patient's condition worsens.

A new nurse is completing an assessment on an 80-year-old patient who is alert and oriented. The patient's daughter is present in the room. Which action by the nurse will require follow-up by the charge nurse? A. The nurse makes eye contact with the patient. B. The nurse speaks only to the patient's daughter. C. The nurse leans forward while talking with the patient. D. The nurse nods periodically while the patient is speaking.

B. The nurse speaks only to the patient's daughter.

The nurse is evaluating whether patient goals and outcomes have been met for a patient with impaired physical mobility due to a fractured leg. Which finding indicates the patient has met an expected outcome? A. The nurse provides assistance while the patient is walking in the hallways. B. The patient is able to ambulate in the hallway with crutches. C. The patient will deny pain while walking in the hallway. D. The patient's level of mobility will improve.

B. The patient is able to ambulate in the hallway with crutches.

After assessing a patient, a nurse develops a standard formal nursing diagnosis. What is the rationale for the nurse's actions? A. To form a language that can be encoded only by nurses B. To distinguish the nurse's role from the physician's role C. To develop clinical judgment based on other's intuition D. To help nurses focus on the scope of medical practice

B. To distinguish the nurse's role from the physician's role

A nurse assigned to an intensive care unit (critical care) is most likely using what type of nursing care delivery model?? A. Team nursing B. Total patient care C. Primary nursing D. Case-management

B. Total patient care

A new nurse notes that the health care unit keeps a listing of patient names in a closed book behind the front desk of the nursing station so patients can be located easily. Which action is most appropriate for the nurse to take? A. Talk with the nurse manager about the listing being a violation of the Health Insurance Portability and Accountability Act (HIPAA). B. Use the book as needed while keeping it away from individuals not involved in patient care. C. Move the book to the upper ledge of the nursing station for easier access. D. Ask the nurse manager to move the book to a more secluded area.

B. Use the book as needed while keeping it away from individuals not involved in patient care.

During a severe respiratory epidemic, the local health care organizations decide to give health care workers priority access to ventilators over other members of the community who also need that resource. Which philosophy would give the strongest support for this decision? A. Deontology B. Utilitarianism C. Ethics of care D. Feminist ethics

B. Utilitarianism

A nurse is evaluating the goal of acceptance of body image in a young teenage girl. Which statement made by the patient is the best indicator of progress toward the goal? A. "I'm worried about what those other girls will think of me." B. "I can't wear dresses that make my hips stick out." C. "I'll wear the blue dress. It matches my eyes." D. "I hope I can go to the pool next summer."

C. "I'll wear the blue dress. It matches my eyes."

A novice nurse asks the preceptor to describe the primary purpose of evaluation. Which statement made by the nursing preceptor is most accurate? A. "An evaluation helps you determine whether all nursing interventions were completed." B. "During evaluation, you determine when to downsize staffing on nursing units." C. "Nurses use evaluation to determine the effectiveness of nursing care." D. "Evaluation eliminates unnecessary paperwork and care planning."

C. "Nurses use evaluation to determine the effectiveness of nursing care."

Which statement by a nurse indicates a good understanding about the differences between data validation and data interpretation? A. "Data interpretation occurs before data validation." B. "Validation involves looking for patterns in professional standards." C. "Validation involves comparing data with other sources for accuracy." D. "Data interpretation involves discovering patterns in professional standards."

C. "Validation involves comparing data with other sources for accuracy."

The patient's son requests to view documentation in the medical record. What is the nurse's best response to this request? A. "I'll be happy to get that for you." B. "You are not allowed to look at it." C. "You will need your mother's permission." D. "I cannot let you see the chart without a doctor's order."

C. "You will need your mother's permission."

A nurse adds the following diagnosis to a patient's care plan: Constipation related to decreased gastrointestinal motility secondary to pain medication administration as evidenced by the patient reporting no bowel movement in seven days, abdominal distention, and abdominal pain. Which element did the nurse write as the defining characteristic? A. Decreased gastrointestinal motility B. Pain medication C. Abdominal distention D. Constipation

C. Abdominal distention

A nurse is discussing quality of life issues with another colleague. Which topic will the nurse acknowledge for increased attention paid to quality of life concerns? A. Health care disparities B. Aging of the population C. Abilities of disabled persons D. Health care financial reform

C. Abilities of disabled persons

A patient presents to the emergency department following a motor vehicle crash that causes a right femur fracture. The leg is stabilized in a full leg cast. Otherwise, the patient has no other major injuries, is in good health, and reports only moderate discomfort. Which is the most pertinent nursing diagnosis the nurse will include in the plan of care? A. Posttrauma syndrome B. Constipation C. Acute pain D. Anxiety

C. Acute pain

A nurse is teaching the staff about the benefits of Nursing Outcomes Classification. Which information should the nurse include in the teaching session? (Select all that apply.) A. Includes seven domains for level 1. B. Uses an easy 3-point Likert scale. C. Adds objectivity to judging a patient's progress. D. Allows choice in which interventions to choose. E. Measures nursing care on a national and international level.

C. Adds objectivity to judging a patient's progress. E. Measures nursing care on a national and international level.

The nurse is caring for a dying patient. Which intervention is considered futile? A. Giving pain medication for pain B. Providing oral care every 5 hours C. Administering the influenza vaccine D. Supporting lower extremities with pillows

C. Administering the influenza vaccine

After reviewing the database, the nurse discovers that the patient's vital signs have not been recorded by the nursing assistive personnel (NAP). Which clinical decision should the nurse make? A. Administer scheduled medications assuming that the NAP would have reported abnormal vital signs. B. Have the patient transported to the radiology department for a scheduled x-ray and review vital signs upon return. C. Ask the NAP to record the patient's vital signs before administering medications. D. Omit the vital signs because the patient is presently in no distress.

C. Ask the NAP to record the patient's vital signs before administering medications.

A patient verbalizes a low pain level of 2 out of 10 but exhibits extreme facial grimacing while moving around in bed. What is the nurse's initial action in response to these observations? A. Proceed to the next patient's room to make rounds. B. Determine the patient does not want any pain medicine. C. Ask the patient what causes the facial grimacing with movement. D. Administer the pain medication ordered for moderate to severe pain

C. Ask the patient what causes the facial grimacing with movement.

Which patient scenario of a surgical patient in pain is most indicative of critical thinking? A. Administering pain-relief medication according to what was given last shift B. Offering pain-relief medication based on the health care provider's orders C. Asking the patient what pain-relief methods, pharmacological and nonpharmacological, have worked in the past D. Explaining to the patient that self-reporting of severe pain is not consistent with the minor procedure that was performed

C. Asking the patient what pain-relief methods, pharmacological and nonpharmacological, have worked in the past

Vital signs for a patient reveal a blood pressure of 187/100. Orders state to notify the health care provider for diastolic blood pressure greater than 90. What is the nurse's first action? A. Follow the clinical protocol for a stroke. B. Review the most recent lab results for the patient's potassium level. C. Assess the patient for other symptoms or problems, and then notify the health care provider. D. Administer an antihypertensive medication from the stock supply, and then notify the health care provider.

C. Assess the patient for other symptoms or problems, and then notify the health care provider.

The nurse is caring for a patient who requires a complex dressing change. While in the patient's room, the nurse decides to change the dressing. Which action will the nurse take just before changing the dressing? A. Gathers and organizes needed supplies. B. Decides on goals and outcomes for the patient. C. Assesses the patient's readiness for the procedure. D. Calls for assistance from another nursing staff member.

C. Assesses the patient's readiness for the procedure.

Which approach will be most appropriate for a nurse to take when faced with the challenge of performing many tasks in one shift? A. Do as much as possible by oneself before seeking assistance from others. B. Evaluate the effectiveness of all tasks when all tasks are completed. C. Complete one task before starting another task. D. Delegate tasks the nurse does not like doing.

C. Complete one task before starting another task.

A nurse is called into the supervisor's office regarding deteriorating work performance since the loss of a spouse 2 years ago. The nurse begins sobbing and says, "I'm falling apart at home as well". Which type of grief is the nurse experiencing? A. Normal grief B. Perceived grief C. Complicated grief D. Disenfranchised grief

C. Complicated grief

1. Which action should the nurse take when using critical thinking to make clinical decisions? A.Makes decisions based on intuition. B. Accepts one established way to provide care. C. Considers what is important in any given situation. D. Reads and follows the heath care provider's orders.

C. Considers what is important in any given situation.

A nurse is working in a facility that has fewer directors which allows for managers and staff to make shared decisions. In which type of organizational structure is the nurse employed? A.Delegation B. Research-based C. Decentralization D. Philosophy of care

C. Decentralization

A patient exhibits the following symptoms: tachycardia, increased thirst, headache, decreased urine output, and increased body temperature. The nurse analyzes the data. Which nursing diagnosis will the nurse assign to the patient? A. Adult failure to thrive B. Hypothermia C. Deficient fluid volume D. Nausea

C. Deficient fluid volume

A charge nurse is evaluating a new nurse's plan of care. Which finding will cause the charge nurse to follow up? A. Assigning a documented nursing diagnosis of Risk for infection for a patient on intravenous (IV) antibiotics B. Completing an interview and physical examination before adding a nursing diagnosis C. Developing nursing diagnoses before completing the database D. Including cultural and religious preferences in the database

C. Developing nursing diagnoses before completing the database

In preparation for the eventual death of a hospice patient, the nurse organizes a meeting of all hospice caregivers. A plan of care to be followed when this patient dies is prepared. Which information will be included in the plan? A. Avoid discussing the death with immediate family members. B. Discuss expectations of normal grieving with the family and friends. C. Encourage the patient's family and friends to verbally express their needs. D. Assure family members they will not be expected to assist with postmortem care

C. Encourage the patient's family and friends to verbally express their needs.

"I know it seems strange, but I feel guilty being pregnant after the death of my son last year," said a woman during her routine obstetrical examination. The nurse spends extra time with this woman, helping her realize bonding with this unborn child will not mean she is replacing the one who died. Which nursing technique does this demonstrate? A. Providing curative therapy B. Promoting spirituality C. Facilitating mourning D. Eradicating grief

C. Facilitating mourning

A patient has a bacterial infection in left lower leg. Which nursing diagnosis will the nurse add to the patient's care plan? A. Infection B. Risk for infection C. Impaired skin integrity D. Staphylococcal leg infection

C. Impaired skin integrity

Which action demonstrates a nurse utilizing reflection to improve clinical decision making? A. Obtains data in an orderly fashion. B. Uses an objective approach in patient situations. C. Improves a plan of care while thinking back on interventions effectiveness. D. Provides evidence-based explanations and research for care of assigned patients.

C. Improves a plan of care while thinking back on interventions effectiveness.

A nurse works full time on the oncology unit at the hospital and works part time on weekends giving immunizations at the local chain pharmacy. While giving an injection on a weekend, the nurse caused injury to the patient's arm and is now being sued. What initial action should the nurse take to initiate an effective legal defense? A. Notify the hospital of the situation to secure legal counsel by the hospital's private attorney. B. Notify the manager of the pharmacy so that the corporation can provide legal counsel. C. Inform the insurance company that is providing one's professional licensure defense insurance. D. Immediately contact the State Board of Nursing to assure protecting the validity of the nursing license.

C. Inform the insurance company that is providing one's professional licensure defense insurance.

Which information concerning a goal indicates a nurse has a good understanding of its purpose? A. It is a statement describing the patient's accomplishments without a time restriction. B. It is a realistic statement predicting any negative responses to treatments. C. It is a broad statement describing a desired change in a patient's behavior. D. It is a measurable change in a patient's physical state.

C. It is a broad statement describing a desired change in a patient's behavior.

In a natural disaster relief facility, the nurse observes that an older-adult male has a recovery plan, while a 25-year-old male is still overwhelmed by the disaster situation. A nurse is planning care for both patients. Which factors will the nurse consider about the different coping reactions? A. Restorative care factors B. Strong financial resource factors C. Maturational and situational factors D. Immaturity and intelligence factors

C. Maturational and situational factors

The nurse is caring for a patient who has an open wound and is evaluating the progress of wound healing. Which priority action will the nurse take? A. Asking the nursing assistive personnel if the wound looks better B. Documenting the progress of wound healing as "better" in the chart C. Measuring the wound and observe for redness, swelling, or drainage D. Leaving the dressing off the wound for easier access and more frequent assessments

C. Measuring the wound and observe for redness, swelling, or drainage

A nurse is teaching the staff about a nursing theory that views a person, family, or community developing a normal line of defense. Which theory is the nurse describing? A. Ego defense model B. Immunity model C. Neuman Systems Model D. Pender's Health Promotion Model

C. Neuman Systems Model

Which staff member does the nurse assign to provide morning care for an older-adult patient who requires assistance with activities of daily living? A. Licensed practical nurse B. Cardiac monitor technician C. Nursing assistive personnel (NAP) D. Another registered nurse on the floor

C. Nursing assistive personnel (NAP)

A nurse is making initial rounds on patients. Which intervention for a patient with poor wound healing should the nurse perform first? A. Reinforce the wound dressing as needed with 4 × 4-inch gauze. B. Perform the ordered dressing change twice daily. C. Observe wound appearance and edges. D. Document wound characteristics.

C. Observe wound appearance and edges.

An adult male reports new-onset, seizure-like activity. An EEG and a neurology consultant's report rule out a seizure disorder. It is determined the patient is using conversion. Which action should the nurse take next? A. Suggest acupuncture. B. Confront the patient on malingering. C. Obtain history of any recent life stressors. D. Recommend a regular exercise program.

C. Obtain history of any recent life stressors.

A novice nurse expresses frustration at not being to complete all interventions for a group of patients in a timely manner. The nurse leaves the rounds report sheets at the nurse's station when caring for patients and reports having to go back and forth between rooms for equipment and supplies. Which type of skill does the nurse need? A. Interpersonal communication B. Clinical decision making C. Organizational D. Evaluation

C. Organizational

A nurse is gathering information about a patient's habits and lifestyle patterns. Which method of data collection will the nurse use that will best obtain this information? A. Carefully review lab results. B. Conduct the physical assessment. C. Perform a thorough nursing health history. D. Prolong the termination phase of the interview.

C. Perform a thorough nursing health history.

Which method of data collection will the nurse use to establish a patient's database? A. Reviewing the current literature to determine evidence-based nursing actions B. Checking orders for diagnostic and laboratory tests C. Performing a physical examination D. Ordering medications

C. Performing a physical examination

A nurse is reviewing a patient's care plan. Which information will the nurse identify as a nursing intervention? A. The patient will ambulate in the hallway twice this shift using crutches correctly. B. Impaired physical mobility related to inability to bear weight on right leg. C. Provide assistance while the patient walks in the hallway twice this shift with crutches. D. The patient is unable to bear weight on right lower extremity.

C. Provide assistance while the patient walks in the hallway twice this shift with crutches.

After assessing the patient and identifying the need for headache relief, the nurse administers acetaminophen. Which action by the nurse is priority for this patient? A. Eliminate headache from the nursing care plan. B. Direct the nursing assistive personnel to ask if the headache is relieved. C. Reassess the patient's pain level in 30 minutes. D. Revise the plan of care.

C. Reassess the patient's pain level in 30 minutes.

The nurse is gathering data on a patient. Which data will the nurse report as objective data? A. States "doesn't feel good." B. Reports a headache. C. Respirations 16. D. Nauseated.

C. Respirations 16.

A nurse is using the critical thinking skill of evaluation. Which action will the nurse take? A. Examine the meaning of data. B. Support findings and conclusions. C. Review the effectiveness of nursing actions. D. Search for links between the data and the nurse's assumptions.

C. Review the effectiveness of nursing actions.

A newly admitted patient who is morbidly obese asks the nurse for assistance to the bathroom. Which action should the nurse take initially? A. Ask for at least two other assistive personnel to come to the room. B. Medicate the patient to alleviate discomfort while ambulating. C. Review the patient's activity orders. D. Offer the patient a walker.

C. Review the patient's activity orders.

1. A nurse preceptor is working with a student nurse. Which behavior by the student nurse will require the nurse preceptor to intervene? A. Reading the patient's plan of care B. Reviewing the patient's medical record C. Sharing patient information with another student D. Documenting medication administered to the patient

C. Sharing patient information with another student

A patient expresses fear of going home and being alone. Vital signs are stable, and the incision is nearly completely healed. What can the nurse infer from the subjective data? A. The patient can now perform the dressing changes without help. B. The patient can begin retaking all of the previous medications. C. The patient is fearful of being discharged. D. The patient's surgery was not successful.

C. The patient is fearful of being discharged.

A charge nurse is reviewing outcome statements written by a novice nurse. The nurse is using the SMART approach. Which patient outcome statement will the charge nurse identify as appropriate to the new nurse? A. The patient will ambulate in hallways. B. The nurse will monitor the patient's heart rhythm continuously this shift. C. The patient will feed self at all mealtimes today without reports of shortness of breath. D. The nurse will administer pain medication every 4 hours to keep the patient free from discomfort.

C. The patient will feed self at all mealtimes today without reports of shortness of breath.

A nurse is planning care for a patient that uses displacement as a defense mechanism. Which information should the nurse consider when planning interventions? A. This copes with stress directly. B. This evaluates an event for its personal meaning. C. This protects against feelings of worthlessness and anxiety. D. This triggers the stress control functions of the medulla oblongata.

C. This protects against feelings of worthlessness and anxiety.

A confused patient with a urinary catheter, nasogastric tube, and intravenous line keeps touching these needed items for care. The nurse has tried to explain to the patient that these lines should not be touched, but the patient continues. Which is the best action by the nurse at this time? A. Apply restraints loosely on the patient's dominant wrist. B. Notify the health care provider that restraints are needed immediately. C. Try other approaches to prevent the patient from touching these care items. D. Allow the patient to pull out lines to prove that the patient needs to be restrained.

C. Try other approaches to prevent the patient from touching these care items.

A nurse agrees with regulations for mandatory immunizations of children. The nurse believes that immunizations prevent diseases as well as prevent spread of the disease to others. Which ethical framework is the nurse using? A. Deontology B. Ethics of care C. Utilitarianism D. Feminist ethics

C. Utilitarianism

1. You are assigned to care for a patient who has just undergone a mastectomy for a malignant tumor. You would most appropriately classify this self-concept component as: A. identity stressor. B. sexuality stressor. C. body image stressor. D. role performance stressor.

C. body image stressor.

cultural respect

Critical to reducing health disparities and improving access to high-quality health care that is respectful and responsive to the needs of the diverse patient

The novice nurse is caring for six patients in this shift. After completing their assessments, the nurse asks where to begin in developing care plans for these patients. Which statement is an appropriate suggestion by the nurse's preceptor? A. "Choose all the interventions and perform them in order of time needed for each one." B. "Make sure you identify the scientific rationale for each intervention first." C. "Decide on goals and outcomes you have chosen for the patients." D. "Begin with the highest priority diagnoses, then select appropriate interventions."

D. "Begin with the highest priority diagnoses, then select appropriate interventions."

A patient cancels a scheduled appointment because they will be attending a Shivah for a family member. Which response by the nurse is best? A. "When families come together for end-of-life decisions, it provides connections." B. "We will reschedule so the appointment does not fall on the Sabbath." C. "Missionary outreach is so important for spiritual comfort." D. "I'm so sorry for your loss."

D. "I'm so sorry for your loss."

A female nursing student in the final term of nursing school is overheard by a nursing faculty member telling another student that she got to insert a nasogastric tube in the emergency department while working as a nursing assistant. Which advice is best for the nursing faculty member to give to the nursing student? A. "Just be careful when you are doing new procedures and make sure you are following directions by the nurse." B. "Review your procedures before you go to work, so you will be prepared to do them if you have a chance." C. "The nurse should not have allowed you to insert the nasogastric tube because something bad could have happened." D. "You are not allowed to perform any procedures other than those in your job description even with the nurse's permission."

D. "You are not allowed to perform any procedures other than those in your job description even with the nurse's permission."

The nurse is caring for a patient supported with a ventilator who has been unresponsive since arrival via ambulance 8 days ago. The patient has not been identified, and no family members have been found. The nurse is concerned about the plan of care regarding maintenance or withdrawal of life support measures. Place the steps the nurse will use to resolve this ethical dilemma in the correct order. 1. The nurse identifies possible solutions or actions to resolve the dilemma. 2. The nurse reviews the medical record, including entries by all health care disciplines, to gather information relevant to this patient's situation. 3. Health care providers use negotiation to redefine the patient's plan of care. 4. The nurse evaluates the plan and revises it with input from other health care providers as necessary. 5. The nurse examines the issue to clarify opinions, values, and facts. 6. The nurse states the problem. 7. Nurse confirms that the problem is ethical in nature A. 6, 7, 1, 2, 5, 4, 3 B. 5, 6, 7, 2, 3, 4, 1 C. 1, 2, 5, 4, 7, 3, 6 D. 7, 2, 5, 6, 1, 3, 4

D. 7, 2, 5, 6, 1, 3, 4

A nurse is prioritizing care for four patients. Which patient should the nurse see first? A. A patient needing teaching about medications B. A patient with a healed abdominal incision C. A patient with a slight temperature D. A patient with difficulty breathing

D. A patient with difficulty breathing

The nurse is evaluating whether a patient's turning schedule was effective in preventing the formation of pressure ulcers. Which finding indicates success of the turning schedule? A. Staff documentation of turning the patient every 2 hours B. Presence of redness only on the heels of the patient C. Patient understands the need for regular turning D. Absence of skin breakdown

D. Absence of skin breakdown

A patient recovering from a leg fracture after a fall reports having dull pain in the affected leg and rates it as a 7 on a 0 to 10 scale. The patient is not able to walk around in the room with crutches because of leg discomfort. Which nursing intervention is priority? A. Assist the patient to walk in the room with crutches. B. Obtain a walker for the patient. C. Consult physical therapy. D. Administer pain medication.

D. Administer pain medication.

Health disparities are unequal burdens of disease morbidity and mortality rates experienced by racial and ethnic groups. These disparities are often exacerbated by: A. bias. B. stereotyping. C. prejudice. D. all of the above.

D. All of the above

While the patient's lower extremity, which is in a cast, is assessed, the patient tells the nurse about an inability to rest at night. The nurse disregards this information, thinking that no correlation has been noted between having a leg cast and developing restless sleep. Which action would have been best for the nurse to take? A.Tell the patient to just focus on the leg and cast right now. B. Document the sleep patterns and information in the patient's chart. C. Explain that a more thorough assessment will be needed next shift. D. Ask the patient about usual sleep patterns and the onset of having difficulty

D. Ask the patient about usual sleep patterns and the onset of having difficulty

The nurse enters a room to find the patient sitting up in bed crying. How will the nurse display a critical thinking attitude in this situation? A. Provide privacy and check on the patient 30 minutes later. B. Set a box of tissues at the patient's bedside before leaving the room. C. Limit visitors while the patient is upset. D. Ask the patient what triggered the crying.

D. Ask the patient what triggered the crying.

A patient's plan of care includes the goal of increasing mobility this shift. As the patient is ambulating to the bathroom at the beginning of the shift, the patient falls. Which initial action will the nurse take next to most effectively revise the plan of care? A. Consult physical therapy. B. Establish a new plan of care. C. Set new priorities for the patient. D. Assess the patient.

D. Assess the patient.

A patient has reduced muscle strength following a left-sided stroke and is at risk for falling. Which intervention is most appropriate for the nursing diagnostic statement Risk for falls? A. Keep all side rails down at all times. B. Encourage patient to remain in bed most of the shift. C. Place patient in room away from the nurses' station if possible. D. Assist patient into and out of bed every 4 hours or as tolerated.

D. Assist patient into and out of bed every 4 hours or as tolerated.

A female teen with celiac disease continues to eat food she knows will make her ill several hours after ingestion. While planning care, the nurse considers maturational and tertiary-level interventions. Which intervention will the nurse add to the care plan? A. Teach the teen about the food pyramid. B. Administer antidiarrheal medications with meals. C. Gently admonish the teen and her parents regarding the consistently poor diet choices. D. Assist the teen in meeting dietary restrictions while eating foods similar to those eaten by her friends.

D. Assist the teen in meeting dietary restrictions while eating foods similar to those eaten by her friends.

A patient's father died a week ago. Both the patient and the patient's spouse talk about the death. The patient's spouse is experiencing headaches and fatigue. The patient is having trouble sleeping, has no appetite, and gets choked up most of the time. How should the nurse interpret these findings as the basis for a follow-up assessment? A. The patient is dying and the spouse is angry. B. The patient is ill and the spouse is malingering. C. Both the patient and the spouse are likely in denial. D. Both the patient and the spouse are likely grieving.

D. Both the patient and the spouse are likely grieving.

A nurse is overseeing the care of patients diagnosed with either severe diabetes or with heart failure. The purpose of this nursing model is to improve cost-effectiveness and quality of care. Which nursing care delivery model is the nurse using? A. Team nursing B. Total patient care C. Primary nursing D. Case-management

D. Case-management

A nurse manager discovers that the readmission rate of hospitalized patients is very high on the hospital unit. The nurse manager desires improved coordination of care and accountability for cost-effective quality care. Which nursing care delivery model is best suited for these needs? A. Team nursing B. Total patient care C. Primary nursing D. Case-management

D. Case-management

A newly hired experienced nurse is preparing to change a patient's abdominal dressing and hasn't done it before at this hospital. Which action by the nurse is best? A. Have another nurse do it so the correct method can be viewed. B. Change the dressing using the method taught in nursing school. C. Ask the patient how the dressing change has been recently done. D. Check the policy and procedure manual for the facility's method.

D. Check the policy and procedure manual for the facility's method.

A nurse is caring for a patient in the last stages of dying. Which finding indicates the nurse needs to prepare the family for death? A. Redness of skin B. Clear-colored urine C. Tense muscles tone D. Cheyne-Stokes breathing

D. Cheyne-Stokes breathing

A palliative team is caring for a dying patient in severe pain. Which action is the priority? A. Provide postmortem care for the patient. B. Support the patient's nurse in grieving. C. Teach the patient the stages of grief. D. Enhance the patient's quality of life.

D. Enhance the patient's quality of life.

An adult who was in a motor vehicle accident is brought into the emergency department by paramedics, who report the following in-transit vital signs:Oral temperature: 99.0° FPulse: 102 beats/min Respiratory rate: 26 breaths/minBlood pressure: 140/106Which hormones should the nurse consider as the most likely causes of the abnormal vital signs? A. ADH and ACTH B. ACTH and epinephrine C. ADH and norepinephrine D. Epinephrine and norepinephrine

D. Epinephrine and norepinephrine

The nurse values autonomy above all other principles. Which patient assignment will the nurse find most difficult to accept? A. Older-adult patient who requires dialysis. B. Teenager in labor who requests epidural anesthesia. C. Middle-aged father of three with an advance directive declining life support. D. Family elder who is making the decisions for a young-adult female member.

D. Family elder who is making the decisions for a young-adult female member.

A nurse must make an ethical decision concerning vulnerable patient populations. Which philosophy of health care ethics would be particularly useful for this nurse? A. Teleology B. Deontology C. Utilitarianism D. Feminist ethics

D. Feminist ethics

A nurse develops a nursing diagnostic statement for a patient with a medical diagnosis of pneumonia with chest x-ray results of lower lobe infiltrates. Which nursing diagnosis did the nurse write? A. Ineffective breathing pattern related to pneumonia B. Risk for infection related to chest x-ray procedure C. Risk for deficient fluid volume related to dehydration D. Impaired gas exchange related to alveolar-capillary membrane changes

D. Impaired gas exchange related to alveolar-capillary membrane changes

A nurse has a transformational leader as a manager. Which finding will the nurse anticipate from working with this leader? A. Increased turnover rate B. Increased patient mortality rate C. Increased rate of medication errors D. Increased level of patient satisfaction

D. Increased level of patient satisfaction

A nurse who is caring for a patient with a pressure ulcer applies the recommended dressing according to hospital policy. Which standard is the nurse following? A. Fairness B. Intellectual standards C. Independent reasoning D. Institutional practice guidelines

D. Institutional practice guidelines

A patient's son decides to stay at the bedside while his father is experiencing confusion. When developing the plan of care for this patient, what should the nurse do to best meet the patient's needs? A. Individualize the care plan only according to the patient's needs. B. Request that the son leave at bedtime, so the patient can rest. C. Suggest that a female member of the family stay with the patient. D. Involve the son in the plan of care as much as possible.

D. Involve the son in the plan of care as much as possible.

A patient diagnosed with terminal cancer asks the nurse what the criteria are for hospice care. Which information should the nurse share with the patient? A. It is for those needing assistance with pain management. B. It is for those having a terminal illness, such as cancer. C. It is for those with completion of an advance directive. D. It is for those expected to live less than 6 months.

D. It is for those expected to live less than 6 months.

A nurse is caring for a dying patient. One of the nurse's goals is to promote dignity and validation of the dying person's life. Which action will the nurse take to best achieve this goal? A. Take pictures of visitors. B. Provide quiet visiting time. C. Call the organ donation coordinator. D. Listen to family stories about the person.

D. Listen to family stories about the person.

A patient was recently diagnosed with pneumonia. The nurse and the patient have established a goal that the patient will not experience shortness of breath with activity in 3 days with an expected outcome of having no secretions present in the lungs in 48 hours. Which evaluative measure will the nurse use to demonstrate progress toward this goal? A. No sputum or cough present in 4 days B. Congestion throughout all lung fields in 2 days C. Shallow, fast respirations 30 breaths per minute in 1 day D. Lungs clear to auscultation following use of inhaler

D. Lungs clear to auscultation following use of inhaler

A charge nurse is supervising the care of a new nurse. Which action by a new nurse indicates the charge nurse needs to intervene? A. Making an ethical clinical decision B. Making an informed clinical decision C. Making a clinical decision in the patient's best interest D. Making a clinical decision based on previous shift assessments

D. Making a clinical decision based on previous shift assessments

A nurse is evaluating goals and expected outcomes for a confused patient. Which finding indicates positive progress toward resolving the confusion? A. Patient wanders halls only at night. B. Patient's side rails are up with bed alarm activated. C. Patient denies pain while ambulating with assistance. D. Patient correctly states names of family members in the room.

D. Patient correctly states names of family members in the room.

Which assessment of a patient who is 1 day post-surgical repair a hip fracture requires immediate nursing intervention? A. Patient ate 40% of clear liquid breakfast. B. Patient's oral temperature is 98.9° F. C. Patient states, "I did not realize I would be so tired after this surgery." D. Patient reports severe pain 30 minutes after receiving pain medication.

D. Patient reports severe pain 30 minutes after receiving pain medication.

The nurse begins a shift assessment by examining a surgical dressing that is saturated with serosanguineous drainage on a patient who had open abdominal surgery yesterday (or 1 day ago). Which type of assessment approach is the nurse using? A. Gordon's Functional Health Patterns B. Activity-exercise pattern assessment C. General to specific assessment D. Problem-focused assessment

D. Problem-focused assessment

A nurse is caring for a patient experiencing stress and is in the evaluation stage of the critical thinking model. Which actions will the nurse take? A. Select nursing interventions and promote patient's adaptation to stress. B. Establish short- and long-term goals with the patient experiencing stress. C. Identify stress management interventions and achieve expected outcomes. D. Reassess patient's stress-related symptoms and compare with expected outcomes.

D. Reassess patient's stress-related symptoms and compare with expected outcomes.

Which action should the nurse take first during the initial phase of implementation? A. Determine patient outcomes and goals. B. Prioritize patient's nursing diagnoses. C. Evaluate interventions. D. Reassess the patient.

D. Reassess the patient.

A nursing assessment for a patient with a spinal cord injury leads to several pertinent nursing diagnoses. Which nursing diagnosis is the highest priority for this patient? A. Risk for impaired skin integrity B. Risk for infection C. Spiritual distress D. Reflex urinary incontinence

D. Reflex urinary incontinence

A nurse is using a critical thinking model to provide care. Which component is first implemented when helping a nurse make clinical decisions? A. Attitude B. Experience C. Nursing process D. Specific knowledge base

D. Specific knowledge base

A registered nurse (RN) is the group leader of licensed practical nurses (LPNs) and nursing assistive personnel (NAP). Which nursing care model is being implemented? A. Case-management B. Total patient care C. Primary nursing D. Team nursing

D. Team nursing

Which action by a nurse indicates application of the critical thinking model to make the best clinical decisions? A. Drawing on past clinical experiences to formulate standardized care plans B. Relying on recall of information from past lectures and textbooks C. Depending on the charge nurse to determine priorities of care D. Using the nursing process

D. Using the nursing process

A nurse is teaching guided imagery to a prenatal class. Which technique did the nurse describe? A. Singing B. Massaging back C. Listening to music D. Visualizing peaceful settings

D. Visualizing peaceful settings

To assess, evaluate, and support a patient's spirituality, the best action a nurse can take is to: A. assist the patient to use faith to get well. B. refer the patient to the health care facility chaplain. C. provide the patient with a variety of religious literature. D. determine the patient's perceptions and belief system.

D. determine the patient's perceptions and belief system.

health disparity

a particular type of health difference that is closely linked with social, economic, and/or environmental disadvantage

Intellectual Background

a person's beliefs about health are shaped in part by the person's knowledge, lack of knowledge, or incorrect information about body functions and illnesses, educational background, and past experiences

Intersectionality

a research and policy model used to study the complexities of people's lives and experiences

self-transcendence

a sense of authentically connecting to one's inner self

A nurse administers an antihypertensive medication to a patient at the scheduled time of 0900. The nursing assistive personnel (NAP) then reports to the nurse that the patient's blood pressure was low when it was taken at 0830. The NAP states that was busy and had not had a chance to tell the nurse yet. The patient begins to complain of feeling dizzy and light-headed. The blood pressure is re-checked, and it has dropped even lower. In which phase of the nursing process did the nurse first make an error? a. Assessment b. Diagnosis c. Implementation d. Evaluation.

a. Assessment

The nurse is reviewing a patient's database for significant changes and discovers that the patient has not voided in over 8 hours. The patient's kidney function lab results are abnormal, and the patient's oral intake has significantly decreased since previous shifts. Which step of the nursing process should the nurse proceed to after this review? a. Diagnosis b. Planning c. Implementation d. Evaluation

a. Diagnosis

A nurse encounters a family who experienced the death of their adult child last year. The parents are talking about the upcoming anniversary of their child's death. The nurse spends time with them discussing their child's life and death. Which nursing principle does the nurse's action best demonstrate? a. Facilitation of normal mourning b. Pain-management technique c. Grief evaluation d. Palliative care

a. Facilitation of normal mourning

The mother of a child who died recently keeps the child's room intact. Family members are encouraging her to redecorate and move forward in life. Which type of grief will the home health nurse recognize the mother is experiencing? a. Normal b. End-of-life c. Abnormal d. Complicated

a. Normal

linguistic competence

ability of caregivers to communicate effectively and convey information in a manner that is easily understood by diverse audiences.

cultural skill

ability to conduct a cultural assessment to collect relevant cultural data regarding the client's presenting problem as well as accurately conducting a culturally-based physical assessment

Examples of marginalized groups

bisexual, gay, transgender, people of color, people who are mentally/physically challenged, people who are not college educated.

9. Nurses have the ethical responsibility of doing or actively promoting good. What is this principle known as? a. Justice b. Veracity c. Beneficence d. Autonomy

c. Beneficence

A nurse is providing nursing care to patients after completing a care plan from nursing diagnoses. In which step of the nursing process is the nurse? a. Assessment b. Planning c. Implementation d. Evaluation

c. Implementation

The nurse performs an intervention for a collaborative problem. Which type of intervention did the nurse perform? a. Dependent b. Independent c. Interdependent d. Physician-initiated

c. Interdependent

2. You are caring for an adolescent patient who underwent a gastric banding procedure 6 months previously. This adolescent tells you, "There is still a fat person inside of me." This type of statement illustrates a flaw in the self-concept of: A. identity. B. self-esteem. C. body image. D. role performance.

c. body image

Meditation

creates relaxation response that reduces daily stress

8. A 60-year-old patient is on several new medications and expresses worry that she will forget to take her pills. Which action by the nurse would be most helpful in this situation? a. Teaching effective coping strategies b. Asking the patient's prescriber to reduce the number of drugs prescribed c. Assuring the patient that she will not forget once she is accustomed to the routine d. Assisting the patient with obtaining and learning to use a calendar or pill container

d. Assisting the patient with obtaining and learning to use a calendar or pill container

1. A nurse determines that the patient's condition has improved and has met expected outcomes. Which step of the nursing process is the nurse exhibiting? a. Assessment b. Planning c. Implementation d. Evaluation

d. Evaluation

A nurse completes a thorough database and carries out nursing interventions based on priority diagnoses. Which action will the nurse take next? a. Assessment b. Planning c. Implementation d. Evaluation

d. Evaluation

10. The nurse is administering medications to the patient who is in renal failure resulting from end-stage renal disease. The nurse is aware that patients with kidney failure would most likely have problems with which pharmacokinetic phase? a. Absorption b. Distribution c. Metabolism d. Excretion

d. Excretion

A young woman who is pregnant with a fetus exposed to multiple teratogens consents to have her fetus undergo serial PUBS (percutaneous umbilical blood sampling) to examine how exposure affects the fetus over time. Although these tests will not improve the fetus's outcomes and will expose it to some risks, the information gathered may help infants in the future. Which ethical principle is at greatest risk? a. Fidelity b. Autonomy c. Beneficence d. Nonmaleficence

d. Nonmaleficence

Health Promotion Model

defines health as a positive, dynamic state, not merely the absence of disease

Internal Variables

developmental stage, intellectual background, emotional factors, spiritual factors

Christianity

discourage use of alcohol and caffeine

role performance

the way in which individuals perceive their ability to carry out significant roles

Perception of Functioning

the way people perceive their physical functioning affects health beliefs and practices

physiological needs

those relating to the basic biological necessities of life: food, drink, rest, and shelter

chronic illness

threatens a person's independence - patient becomes anxious, fear and powerlessness.

code of ethics

- A statement of philosophical ideals of right and wrong that define the principles you will use to provide care to your patients. -Ask yourself how your ethics, values, and practice compares with established standards. - "The Code of Ethics for Nurses with Interpretive Statements" is a guide for carrying out nursing responsibilities and providing quality nursing care.

Implementation

- Acute care: Support systems,Diet therapies,Supporting rituals - Restorative/continuing care: Prayer,Meditation,Supporting grief work

Spiritual Health

- Spiritual health represents a balance. - Spiritual health matures with increasing awareness of meaning, purpose, and life values. - Spiritual beliefs change as patients grow and develop.

Spirituality

An awareness of one's inner self and a sense of connection to a higher being, nature, or some purpose greater than oneself - Includes personal beliefs that help a person maintain hope and get through difficult situations. - The human spirit is powerful, and spirituality has different meanings for different people. - Nurses need to be aware of their own spirituality to provide appropriate and relevant spiritual care to others.

A nurse is documenting end-of-life care. Which information will the nurse include in the patient's electronic medical record? (Select all that apply.) A. Reason for the death B. Time and date of death C. Special preparations of the body D. Location of body identification tags E. Time of body transfer and destination

B,D,E

A pediatric oncology nurse floats to an orthopedic trauma unit. Which action should the nurse manager of the orthopedic unit take to enable safe care to be given by this nurse? A. Provide a complete orientation to the functioning of the entire unit. B. Determine patient acuity and care the nurse can safely provide. C. Allow the nurse to choose which mealtime works best. D. Assign nursing assistive personnel to assist with care.

B. Determine patient acuity and care the nurse can safely provide.

External Variables

External variables influencing a person's health beliefs and practices include family practices, psychosocial and socioeconomic factors, and cultural background.

FICA assessment tool

F - faith, belief, meaning I - importance and influence C - community A - address(interventions)

tertiary prevention

Occurs when a defect or disability is permanent or irreversible. Involves minimizing effects of long term disease.

role performance stressor

Role conflict (ex-middle aged women with kids assuming responsibility of her parents), Role ambiguity (ex- pressure adolescents to assume adult-life roles) Role strain (unsuited to a role) providing care to family member with dementia) Role overload (having more roles or responsibilities than manageable)

atheist

a person who believes there is no god

positive health behaviors

activities related to maintaining, attaining, or regaining good health and preventing illness

teach back method

allows patient to repeat back key concepts from teaching session to confirm understanding

faith

allows people to have firm beliefs despite lack of physical evidence

A staff member verbalizes satisfaction in working on a particular nursing unit because of the freedom of choice and responsibility for the choices. This nurse highly values which element of shared decision making? a. Authority b. Autonomy c. Responsibility d. Accountability

b. Autonomy

The nurse questions a health care provider's decision to not tell the patient about a cancer diagnosis. Which ethical principle is the nurse trying to uphold for the patient? a. Consequentialism b. Autonomy c. Fidelity d. Justice

b. Autonomy

25. Which entry will require follow-up by the nurse manager? 0800 Patient states, "Fell out of bed." Patient found lying by bed on the floor. Legs equal in length bilaterally with no distortion, pedal pulses strong, leg strength equal and strong, no bruising or bleeding. Neuro checks within normal limits. States, "Did not pass out." Assisted back to bed. Nurse call system within reach. Bed monitor on.-------------------Jane More, RN0810 Notified primary care provider of patient's status. New orders received. -------------------Jane More, RN0815 Portable x-ray of L hip taken in room. States, "I feel fine." -------------------Jane More, RN0830 Incident report completed and placed on chart.-------------------Jane More, RN a. 0800 b. 0810 c. 0815 d. 0830

d. 0830

Health belief model third component

likelihood that a person will take action- arises from the perception of benefits or and barriers to taking action.

culturally congruent care or transcultural care

provide care based on patients individual cultural beliefs, practices and values. communication is important.

acute illness

the sudden illness/injury that creates an unanticipated scramble to integrate and cope with new realities

Religion

the system of organized beliefs and worship that a person practices to outwardly express spirituality.


Kaugnay na mga set ng pag-aaral

AWS Cloud Practitioner Exam - Official Practice Sets

View Set

MUCO 120 lecture quizzes- Quiz 5

View Set

Intro to food & nutrition Ch. 11

View Set

Ch. 29: Growth and Development of the Adolescent

View Set